Sei sulla pagina 1di 72

Contenido

Contenido...........................................................................................................................1 DAZINGER.......................................................................................................................3 HARR..............................................................................................................................5 DUALISMO (CHURCHLAND).......................................................................................7 DUALISMO (CHURCHLAND).....................................................................................10 DUALISMO....................................................................................................................1 DUALISMO (LO!E).....................................................................................................1" CLAUSURA CAUSAL DE LO #$SICO (LO!E).........................................................1% CONDUC&ISMO............................................................................................................1' CONDUC&ISMO .........................................................................................................1' CONDUC&ISMO 3.........................................................................................................1( &EOR$AS DE LA IDEN&IDAD ) (SEARLE)............................................................... 0 MA&ERIALISMO (SEARLE)........................................................................................ #UNCIONALISMO (CHURCHLAND)......................................................................... " MA&ERIALISMO ELIMINA&I*O (CHURCHLAND)................................................ 5 &ESIS DE LA AU&ONOM$A (#LANAGAN)............................................................... % +RO,LEMA ON&OL-GICO........................................................................................ 7 +RO,LEMA ON&OL-GICO ..................................................................................... ' +RO,LEMA ON&OL-GICO 3..................................................................................... ( LA MEN&E RE+RESEN&ACIONAL (+ERNER ) CA+. 1).........................................3 LA MEN&E RE+RESEN&ACIONAL (+ERNER ) CA+. ).........................................3" LA MEN&E RE+RESEN&ACIONAL (+ERNER).........................................................35 LA MEN&E RE+RESEN&ACIONAL (+ERNER).........................................................3% IN&ENCIONALIDAD (SEARLE).................................................................................37 +RO,LEMA SEM.N&ICO...........................................................................................3' +RO,LEMA SEM.N&ICO (CHURCHLAND)............................................................"0 IN&ENCIONALIDAD / CHURCHLAND....................................................................."1 ........................................................................................................................................" CON&ENIDO MEN&AL (LO!E)................................................................................."3 CON&ENIDO MEN&AL (LO!E) / ............................................................................"" SENSACI-N 0 A+ARIENCIA (LO!E)......................................................................"% SENSACI-N 0 A+ARIENCIA (LO!E) / ................................................................."7 SENSACI-N 0 +ERCE+CI-N (LO!E)......................................................................"( +ERCE+CI-N (LO!E)..................................................................................................5 +ERCE+CI-N (SEARLE)..............................................................................................55 +ENSAMIEN&O 0 LENGUA1E (LO!E)....................................................................5% +ENSAMIEN&O 0 LENGUA1E (LO!E) / ...............................................................57

+ENSAMIEN&O 0 LENGUA1E (LO!E) / 3...............................................................5( +ENSAMIEN&O 0 LENGUA1E (LO!E) ) "..............................................................%1 RACIONALIDAD HUMANA E IA (LO!E 0 CHURCHLAND)...............................% RACIONALIDAD HUMANA (LO!E)........................................................................%5 IN&ELIGENCIA AR&I#ICIAL (LO!E).......................................................................%7 RACIONALIDAD E IN&ELIGENCIA AR&I#ICIAL...................................................%' ACCI-N2 LI,RE AL,EDR$O E INCONCIEN&E (SEARLE 3(0 0 30")....................70 +RO,LEMA E+IS&EMOL-GICO (CHURCHLAND 0 SEARLE)............................71

DAZINGER
1) En los primeros aos de la psicologa experimental aparecen dos modelos de experimento psicolgico diferentes: a. El modelo de Leipzig y el modelo del experimento clnico b. El modelo psicolgico y el modelo hipntico c. El modelo psicofsico y el modelo mdico ) En el modelo de !eip"ig: a. Los roles de conductor de la investigacin y sujeto del experimento eran estrictamente fijados b. Los participantes podan asumir tanto el rol de conductor del experimento como el de sujeto de experimentacin c. El rol de conductor de la investigacin era m s importante !ue el de sujeto de experimentacin #) En el modelo del experimento clnico $%ars): a. Los participantes podan asumir tanto el rol de conductor del experimento como el de sujeto de experimentacin b. Los roles de conductor de la investigacin y sujeto del experimento eran estrictamente fijados y no intercambiables c. El sujeto del experimento era el !ue diriga la experimentacin. &) 'tro modelo de experimentacin ()e emerge a fines del siglo *I* es el de +rancis Galton, en Inglaterra- Entre s)s caractersticas m.s so/resalientes se seala: a. El diagnstico y tratamiento de las enfermedades psicolgicas b. La medicin de las facultades mentales de los solicitantes a sus servicios. c. La medicin de la inteligencia de los sujetos del experimento 0) Elegir el rasgo com1n a los tres modelos de experimentacin: a. La importancia dada a los factores sociales en la medicin de las aptitudes individuales. b. El objeto de investigacin se localizara en la interioridad de un individuo considerado aisladamente" es decir" desvinculado de las condiciones sociales. c. El inters por diagnosticar las enfermedades mentales para su posterior tratamiento psicolgico. 2) 3Generalmente los in4estigadores 5 los s)6etos 7acan extensi4o a la sit)acin experimental el patrn 5a existente de relacin m8dico9 paciente: ;<on c).l de los modelos de experimentacin reseados se p)ede identificar esta afirmacin= a. El modelo de experimentacin de #alton" $nglaterra. b. El modelo de experimentacin de %undt" en Leipzig. c. El modelo de experimentacin de &ars. >) 3!os in4estigadores 5 s)s s)6etos eran extraos ()e se re)nan 5 acorda/an cola/orar d)rante el /re4e perodo de la experimentacin o del test: ;<on c).l de los modelos de experimentacin reseados se p)ede identificar esta afirmacin= a. El modelo de experimentacin de %undt" en Leipzig. b. El modelo de experimentacin de &ars. c. El modelo de experimentacin de #alton" $nglaterra. ?) 3!os participantes $del experimento) eran colegas in4estigadores, con intereses com)nes 5 )na amistad ()e a men)do a/arca/a 4arios experimentos: ;<on c).l de los modelos de experimentacin reseados se p)ede identificar esta afirmacin= a. El modelo de experimentacin de %undt" en Leipzig. b. El modelo de experimentacin de #alton" $nglaterra.

c. El modelo de experimentacin de &ars. @) El inter8s por el conocimiento de los procesos elementales de la mente 7)mana normal, es el o/6eti4o principal del modelo de experimentacin a. 'el modelo clnico de &ars. b. 'e #alton" en $nglaterra. c. 'e %undt" en Leipzig. 1A) El inter8s por el conocimiento del desempeo comparado de indi4id)os, es el o/6eti4o principal del modelo de experimentacin a. 'e %undt" en Leipzig. b. 'el modelo clnico de &ars. c. 'e #alton" en $nglaterra. 11) El inter8s en el conocimiento de los estados patolgicos, es el o/6eti4o principal del modelo de experimentacin a. 'e #alton" en $nglaterra. b. 'el modelo clnico de &ars. c. 'e %undt" en Leipzig.

BARRC
1) Deg1n Barr8 la psicologa cogniti4a es )na ciencia 7/rida 5a ()e de/era incl)ir: a. $nteligencia artificial y modelos computacionales de lo mental. b. (n estudio naturalista de los fenmenos psicolgicos cultural e histricamente diversos" manifiestos en lo !ue las personas hacen. c. )odas las otras son correctas. d. (na investigacin emprica y terica de los mecanismos neuronales por los !ue las personas act*an y piensan. ) El atomismo como pres)p)esto ontolgico implica $m.s de )na): a. El determinismo. b. La independencia lgica de las partculas. c. El indeterminismo. d. La dependencia lgica de los centros de actividad. #) Deg1n Barr8 en psicologa cogniti4a, como en las ciencias nat)rales, los modelos explicati4os son necesarios para: a. +ubsumir casos concretos bajo leyes naturales universales. b. Clasificar los fenmenos a estudiar" logrando taxonomas !ue simplifi!uen el dominio de la psicologa. c. ,cceder" por analoga" a las causas inobservables !ue expli!uen los fenmenos observables. &) Deg1n Barr8 en psicologa cogniti4a, como en las ciencias nat)rales, los modelos analticos son necesarios para: a. +ubsumir casos concretos bajo leyes naturales universales. b. $maginar las causas inobservables de los fenmenos observables. c. +implificar la complejidad observada en un determinado fenmeno para poder estudiarlo mejor. 0) !a o/ser4acin de la acti4acin de di4ersas "onas cere/rales es )n e6emplo de los fenmenos de la regin: a. (no. b. )res. c. 'os. 2) Deg1n Barr8, 7a5 posi/ilidad de acceso a los fenmenos de la regin tres: a. +" a travs de la observacin directa. b. +" a travs de modelos explicativos. c. -o" por!ue no son observables. >) Deg1n Barr8, los sistemas de clasificacin: a. .ijan el mbito de fenmenos !ue definen el dominio de investigacin de cada una de las ciencias y de este modo expresan ontologas. b. +on naturales y nos son dados por la observacin de las entidades investigadas por una determinada ciencia. c. +on nominales" por lo tanto slo suponen una clasificacin simblica desvinculada de la esencia real de los fenmenos a estudiar. ?) Deg1n Barr8, )n modelo: a. Es una creacin libre imaginada para generar diversas im genes de los fenmenos. b. Es una explicacin !ue apela a la aplicacin de leyes a un caso concreto. c. Es una representacin an loga de un sistema" para alg*n propsito determinado" construido mediante el uso de la imaginacin.

@) Deg1n Barr8, a)n modelo analtico se le exige ()e sea $m.s de )na): a. Lo m s simple posible. b. Claro y frtil. c. ,bstracto e idealizado" aun!ue manteniendo cierto grado de fidelidad con los fenmenos observados. d. Lgicamente consistente. 1A) Ena red ne)ronal artificial constit)ira )n e6emplo de: a. /odelo explicativo. b. /odelo analtico. c. -inguno de los dos. d. ,mbos. 11) En )n modelo explicati4o, seg1n Barr8, la f)ente 5 el o/6eto: a. Coinciden b. Es indiferente. c. -o coinciden. 1 ) En )n modelo analtico, seg1n Barr8, la f)ente 5 el o/6eto: a. #eneralmente coinciden" pero hay excepciones. b. +iempre coinciden. c. Es indiferente. d. -o coinciden. 1#) Deg1n Barr8, a)n modelo explicati4o se le exige ()e sea: a. +imple. b. (na copia fiel de los fenmenos !ue explica. c. Claro y frtil. d. ,pto para la manipulacin eficaz de entidades inobservables. 1&) Deg1n Barr8, ;()8 reperc)siones t)4o el positi4ismo so/re la psicologa= a. ,compa0 el surgimiento del conductismo en psicologa. b. ,compa0 el surgimiento de la psicologa experimental 1undtiana. c. ,compa0 el surgimiento de la psicologa cognitiva d. -o tuvo mayores repercusiones. 10) !os positi4istas no aceptaran las esencias reales para el desarrollo de taxonomas cientficas por()e $m.s de )na): a. -unca podra probarse su existencia m s all de toda duda. b. -inguna de las anteriores es correcta. c. ,un!ue son observables" no cumplen un rol importante para las taxonomas. d. #eneralmente son tericas e inobservables.

DEA!IDF' $<BER<B!AND)
1) El d)alismo cartesiano 7ace )na caracteri"acin del c)erpo como substancia extensa 5 de la mente como substancia pensante- D)s pro/lemas son acerca de la caracteri"acin de am/os 5 de s) interrelacin- GEn s) caracteri"acin de la D)/stancia %ensante, Descartes seala ()e: a2 La sustancia pensante tiene algunas caractersticas !ue comparte con la materia" y !ue le permiten sentir" decidir acerca de acciones motivadas por deseos y creencias" etc. b2 La sustancia pensante posee todas las caractersticas !ue tiene la materia" pero se le a0aden otras !ue la hacen ontolgicamente diferente" como su no3localidad" su indivisibilidad3simplicidad" etc. c2 La sustancia pensante no tiene las caractersticas espaciales de la materia" pero interact*an con esta. ) Deg1n el D)alismo %op)lar: a2 La persona es su mente y no tiene interacciones con su cuerpo. b2 La persona es una m !uina operada por un fantasma. c2 La persona es un fantasma dentro de una m !uina. #) El D)alismo %op)lar: a2 es un 'ualismo +ubstancial b2 no es un 'ualismo +ubstancial sino de &ropiedades &) !a mente, para el D)alismo %op)lar--a2 Es una substancia cuyas propiedades mentales le permiten la interaccin con las propiedades materiales" !ue son exclusivas del cuerpo. b2 es una substancia con propiedades tanto mentales como espaciales" y ocupa un lugar en el cuerpo. 0) El D)alismo %op)lar es compati/le ontolgicamente con alg)nas teoras fsicas contempor.neas, p)esto ()e: a2 ,l postular !ue la mente posee slo propiedades no materiales" respeta el mbito de propiedades fsicas de !ue tratan las teoras fsicas contempor neas. b2 al postular !ue es una substancia energtica" hace explicable la interaccin mente3cuerpo" y esta interaccin" al ser causal" cae bajo las leyes de conservacin del movimiento y la energa 2) !a posicin ontolgica llamada Dualismo de Propiedades admite ()e el cere/ro tiene tanto propiedades fsicas como no fsicas $sentir dolor, perci/ir color, pensar, desear---)- Estas propiedades son las ()e atri/)imos a la inteligencia consciente, 5 se dice ()e son no-fsicas p)esto ()e: a2 aun!ue algunos cientficos pueden reducirlas a trminos fsicos conocidos" sin embargo fallan a la hora de dar una explicacin de ellas. b2 son irreductibles" aun!ue permiten alguna explicacin parcial en trminos fsicos. c2 son irreductibles o inexplicables en trminos fsicos conocidos. >) Ba5 )na posicin ()e es )n modo del dualismo de propiedades, 5 ()e se caracteri"a por sostener ()e los fenmenos mentales no forman parte de los fenmenos fsicos del cere/ro, pero ()e son estos fenmenos fsicos del cere/ro ()ienes determinan acciones 5 cond)ctas en )n indi4id)o- Esta posicin ontolgica se denomina: a2 Epifenomenismo b2 Emergentismo /onista c2 'ualismo de )abla de / !uina d2 'ualismo de propiedades elementales

?) Deg1n el epifenomenismo: a2 Los estados mentales son actividades cerebrales pero no ejercen ninguna accin causal sobre el mundo fsico externo" aun!ue s sobre nuestro cuerpo. b2 los estados mentales son actividades cerebrales !ue interact*an causalmente con el cuerpo y con el mundo fsico. c2 Los estados mentales son producto de actividades cerebrales !ue no ejercen ninguna accin causal ni sobre el mundo fsico externo" ni sobre nuestro cuerpo. @) %)esto ()e el Epifenomenismo acepta ()e el cere/ro e6erce dos lneas de ca)salidad: )na c)ando ca)sa los epifenmenos llamados estados mentales, 5 otra c)ando ca)sa deterministamente las acciones 5 cond)ctas del c)erpoH entonces: a2 El Epifenomenismo no es compatible ni con un fisicismo 4de conductas o acciones2" ni con un dualismo de substancias. b2 El Epifenomenismo es compatible con un dualismo substancial dado la diversidad de propiedades. c2 el Epifenomenismo es compatible con un fisicismo respecto de conductas y acciones. 1A) El Epifenomenismo de6a c)estiones sin resol4er, como por e6emplo, ;cmo p)ede 7a/lar de identidad entre epifenmeno mental 5 estado cere/ral fsico, si tienen distintas propiedades=H o sea, cmo p)ede )n estado fsico cere/ral ca)sar efectos c)5as propiedades no p)eden red)cirse a las propiedades de dic7o estado fsico cere/ral, ni si()iera explicati4amente= !a preg)nta es: ;)na forma de D)alismo de %ropiedades pero interaccionista tiene )na resp)esta para esta c)estin= a2 +i. b2 no. 11) El D)alismo Interaccionista de %ropiedades sostiene por ello ()e: a2 las propiedades mentales tienen efectos causales sobre el cerebro y sobre la conducta. b2 Las propiedades mentales son causadas por el cerebro pero son reductibles a las propiedades fsicas del cerebro. c2 Las propiedades fsicas del cerebro son suficientes para explicar la conducta y las acciones. 1 ) Es as ()e el cere/ro, seg1n este tipo de D)alismo, tiene propiedades tanto mentales como fsicas, 5 6)ntas determinan las acciones- Esto permite decir ()e: a2 nuestros deseos y actos voluntarios no interaccionan con nuestra conducta" aun!ue bastan para explicarlas b2 nuestros deseos y actos voluntarios" como estados mentales" son la causa de nuestra conducta. 1#) %or ello, otro D)alismo de %ropiedades sostiene ()e las propiedades mentales ()e manifiesta el cere/ro 7an estado siempre presentes en la materia- A este d)alismo se lo llama: a2 'ualismo de &ropiedades Elementales b2 'ualismo &opular de +ubstancias c2 'ualismo Emergentista de &ropiedades 1&) Este D)alismo de %ropiedades Elementales, a)n()e res)el4e alg)nos pro/lemas del Emergentismo, tiene otros pro/lemas a la 7ora de dar c)enta de: a2 por !u" si las propiedades son tan b sicas o elementales" han aparecido recin despus de tantos millones de a0os de evolucin de la materia5

b2 por !u" si las propiedades mentales nos resultan tan cognoscibles a nosotros" !ue las experimentamos" resultan imposibles de constatar en cual!uier otro ser material" incluso nuestros congneres5 c2 por !u" si las propiedades son b sicas" no se las constata en cada parte de materia como por ejemplo" un electrn" un tomo o una molcula de cual!uier substancia5

DEA!IDF' $<BER<B!AND)
1) <7)rc7land, entre otros a)tores ()e 4eremos, presenta s)cintamente alg)nos arg)mentos a fa4or 5 en contra de las posiciones d)alistas acerca de lo mental- En arg)mento a fa4or del D)alismo es el religioso- Este arg)mento dice ()e, p)esto ()e el s)6eto religioso pide ()e exista )na co7erencia entre creencia religiosa 5 la idea de ()e n)estra alma es inmortal, entonces de/e ser 4erdad ()e el alma es inmortal y distinta del cuerpo, p)esto ()e este 1ltimo es, e4identemente, mortal- Deale la o/6ecin ()e se plantea a este arg)mento: a2 Las razones !ue presenta una religin determinada en ocasiones se contradicen con las !ue presenta otra religin" y" salvo cada uno !ue est convencido de su propia religin" un observador externo no tendr buenos argumentos para sostener lo !ue dice una frente a lo !ue dice otra. b2 -o todas las religiones sostienen o sostuvieron la inmortalidad del alma. ) El Argumento de la introspeccin ()e presenta <7)rc7land sostiene ()e los contenidos de n)estra conciencia--a2 no nos muestran el flujo de estados conscientes y propiedades mentales" sino la actividad electro!umica de nuestras redes neuronales. b2 no nos muestran la actividad electro!umica de nuestras redes neuronales sino el flujo de estados conscientes y propiedades mentales. #) Di sost)4i8semos como 4.lido el arg)mento de la Introspeccin, 7a/ra ()e afrontar otro pro/lema ) o/6ecin al mismo- <omo dice el arg)mento, c)ando 7acemos introspeccin, tenemos conciencia del fl)6o de estados mentales, pensamientos, deseos, sensaciones, etc-, 5 no tenemos conciencia de ()e n)estras ne)ronas est.n reali"ando enlaces ()micos entre ellas- %ero entonces de/eramos sostener ()e: a2 si de hecho slo tenemos conciencia de estos estados mentales como distintos de los estados neurofisiolgicos" entonces estos *ltimos no existen o no tienen nada !ue ver con los mentales. b2 si de hecho hay actividad neuro!umica en mi cerebro" entonces la introspeccin no nos muestra las cosas como son en su naturaleza ntima &) En arg)mento similar al anterior es el llamado Argumento de la Irreductibilidad- Este dice ()e: a2 los estados neurofisiolgicos !ue ocurren en nuestro cerebro 4conexiones !umicas entre neuronas" etc.2 no pueden dar cuenta de los llamados estados mentales" ni tampoco bastan para explicarlos. &or ello es !ue sostenemos !ue tales estados mentales no existen. b2 las aptitudes mentales 4razonamiento" estados cualitativos2" y el contenido significativo de los pensamientos y creencias" no parecen ser reductibles a alg*n sistema fsico !ue !uiera emularlos" predecirlos o producirlos. 0) El Argumento de la Irreductibilidad, como arg)mento a fa4or del D)alismo, tiene 4arios pro/lemas- Deale cul de los siguientes no sera un problema de este argumento, seg1n <7)rc7landa2 ,un!ue un sistema fsico todava no pueda reducir a lo fsico la significatividad de lo mental" eso no implica !ue no sea posible en alg*n momento. ,dem s" tampoco los dualistas sustanciales pueden explicar el hecho de la significatividad. En este sentido" tanto el materialismo como el dualismo fallan en la explicacin de la significatividad. b2 La complejidad del lenguaje humano parece irreproducible en lenguaje de ordenador" pero investigaciones empricas en inteligencia artificial y gram tica generativa 4Choms6y2 sugieren !ue el uso del lenguaje no es un atributo exclusivo de lo mental" y !ue en alg*n momento un sistema fsico podr usarlo. c2 &uesto !ue no existe todava ning*n sistema fsico 4inform tico o mec nico2 !ue pueda emular las aptitudes mentales 4procesamiento de informacin" c lculo" capacidad de decisin en mbitos complejos2" es de suponer !ue no existir jam s esa simulacin" atendiendo al hecho de !ue" en la actualidad" la complejidad !ue estos sistemas inform ticos han alcanzado es ya muy alta.

d2 +i se dice !ue los fenmenos parapsicolgicos son irreductibles a la explicacin fsica" eso no invalida !ue el da de ma0ana sean explicados fsicamente. e2 existen objetos con soporte fsico !ue pueden perfectamente desempe0ar procesos de razonamiento matem tico con mayor precisin y rapidez !ue el hombre. 2) Ieamos a7ora alg)nos arg)mentos contra el Dualismo, ()e presenta <7)rc7land- Atienda a ()e estos arg)mentos son de tipo Materialistas- El primero es el Argumento de la simplicidad- Este dice ()e: a2 El materialismo" si bien no explica satisfactoriamente todo fenmeno psicolgico o mental" ofrece una teora m s simple ontolgicamente hablando 4o sea" en cuanto a lo !ue postula como entidades del mundo2. &or otro lado" el dualismo tampoco logra explicar todo lo !ue se propone respecto a lo mental. &or ello" el materialismo es preferible al dualismo. b2 La materia de !ue est hecho el cuerpo es m s simple !ue la substancia mental" puesto !ue es m s comprensible para nosotros lo material !ue lo espiritual. c2 El dualismo puede explicar la accin mediadora del cerebro" como lo ha hecho 'escartes apelando a la funcin de la gl ndula pineal" pero el materialismo" por ser m s simple en lo explicativo" sigue siendo la opcin preferible" aun!ue no nos expli!ue la funcin mediadora del cerebro para explicar nuestras conductas >) En arg)mento similar al anterior es el llamado De la Impotencia Explicati a- En las opciones ()e se sig)en, todas son ra"ones para sostener ()e el materialismo es me6or ()e el d)alismo, excepto )na- ;<).l es esa= a2 El dualismo no ofreci una explicacin de la sustancia mental" su conexin con el cuerpo" funcionamiento" etc. b2 El dualismo" si bien no ofrece una buena explicacin de la sustancia corporal" s nos ofrece una explicacin satisfactoria de la sustancia mental" puesto !ue es lo !ue se propone hacer. c2 El materialismo tiene una teora bien formulada sobre la relacin entre el cerebro" el sistema nervioso central y sus funciones respecto de la conducta. d2 El materialismo muestra mejor el vnculo entre fenmenos mentales y cuestiones fisiolgicas.

DEA!IDF'
1 Deg1n <7)rc7land el enfo()e d)alista de la mente a/arca 4arias teoras m)5 diferentes, pero todas coinciden en ()e la esencia de la inteligencia consciente reside en algo noJ !a afirmacin caracterstica del d)alismo cartesiano es ()e cada mente esJ # El representante m.s importante del d)alismo s)stancial 7a sidoJ & !oKe sostiene ()e las resp)estas ()e intentemos dar a c)estiones como si la mente es distinta del c)erpo o de alg)na de s)s partes, el cere/ro por e6emplo, 5 de si la mente posee propiedades, como la consciencia, ()e son excl)si4amente s)5asJ 0 !oKe s)giere )sar como t8rmino general 1nico ()e a/ar()e a todas las cosas dotadas de mente, reales o posi/les, al t8rminoJ 2 El est)dio de las categoras generales de las cosas ()e existen o p)eden existir, es )na parte de los est)dios filosficos ()e se denominaJ > Ena s)/stancia se caracteri"a para Descartes por )na propiedad /.sica de la ()e no p)ede carecer 5 seg)ir siendo con todo la mismaJ ? En contraste con la s)/stancia fsica, Descartes consider ()e la propiedad definitoria de la s)/stancia mental era elJ @ Deg1n <7)rc7land a)n()e el d)alismo no es la concepcin m.s dif)ndida dentro de la com)nidad filosfica 5 cientfica act)al es la teora m.s pop)lari"ada so/re la mente- En opinin de este a)tor esto se de/era a: a. La influencia de las religiones universales b. 7ue ha sido una de las hiptesis dominantes en el pensamiento occidental 1A %ara !oKe c)ando decimos ()e las personas Ltienen mentesL estamos, con seg)ridad, diciendo a. algo acerca de ciertas 8cosas8 !ue las personas" de alguna manera" poseen b. algo acerca de las propiedades de las personas 11 %ara !oKe existen dos formas posi/les de d)alismo s)stancial: en la primera se considera a la persona 7)mana como )na s)stancia inmaterial ()e se enc)entra en relacin con )n c)erpo 5 ()e carece de c)al()ier propiedad fsica- !a seg)nda a. ninguno b. los dos c. el primero d. el segundo 1 ;M)8 ra"ones tena Descartes para sostener esta, al parecer, tan extraa 4isin de nosotros mismos= a. consideraba !ue nuestros cuerpos eran simplemente incapaces de realizar actividades inteligentes por s mismos incapaces de pensar. b. crea !ue la conducta de los cuerpos" abandonados a s mismos" estaba completamente gobernada por leyes mec nicas !ue determinaban sus movimientos como puros efectos de los movimientos de los otros cuerpos !ue entraran en contacto con ellos. 1# !as dific)ltades ()e plantea/a el d)alismo cartesiano lle4aron a ()e se pensara en )na forma menos extrema de d)alismo s)stancial, ()e es la ()e encontramos en )na concepcin a la ()e <7)rc7land denomino d)alismo pop)lar- Deg1n esta teora a. la m !uina es el cuerpo humano

b. el fantasma es una sustancia espiritual c. una persona es literalmente un 9fantasma dentro de una m !uina9 d. constitucin interna de la sustancia espiritual es totalmente diferente de la materia fsica pero aun as posee plenamente las propiedades espaciales 1& !as ideas /.sicas de las teoras ()e se agr)pan /a6o la designacin de d)alismo de propiedades sostienen ()e, si /ien a() no 7a5 ()e considerar ning)na s)stancia f)era del cere/ro, ()e es algo fsico: a. ste tiene un conjunto especfico de propiedades !ue no posee ning*n otro b. son las propiedades caractersticas de la inteligencia consciente c. son no fsicas en el sentido de !ue jam s podran reducirse a los conceptos de las ciencias fsicas conocidas ni podran ser explicadas en esos trminos d. estas propiedades especiales son lo no fsico

DEA!IDF' $!'NE)
La perspectiva de Lo1e" al analizar la postura 'ualista" es algo diferente de la planteada por Churchland. +i bien presenta los argumentos de 'escartes y su dualismo substancial" Lo1e define los conceptos !ue utilizar en su discusin de manera !ue nos conduce a ver sus argumentos a favor de la posicin dualista substancial no3cartesiana !ue l sostiene. En primer lugar" dir !ue el problema no es cmo la mente se relaciona con el cuerpo sino m s bien cmo los seres de experiencia nos relacionamos con nuestros cuerpos. , estos seres de experiencia le llamar en m*ltiples ocasiones como personas. 1) Ena posicin d)alista n)nca dira entonces ()e: a2 la persona es una entidad compuesta una de cuyas partes es su cuerpo" y la otra su espritu o alma inmaterial b2 la persona es algo completamente separado de su cuerpo c2 la persona es idntica a su cuerpo" o a alguna parte destacada de ste 4por ejemplo" el cerebro2. ) Entre los arg)mentos ()e Descartes presenta a fa4or de s) posicin d)alista est.n: a2 el argumento de la simplicidad explicativa. b2 el argumento de clausura causal. c2 el argumento de lo concebible. #) En el Arg)mento de lo <once/i/le Descartes sostiene ()e, p)esto ()e conci/e ()e es co7erente pensarse como existiendo sin poseer c)erpo alg)no, entonces parece ()e 8l $s) 5o) no es id8ntico a s) c)erpo- En este arg)mento, todo radica en: a2 el imaginarme !ue un estado de cosas es tal" es suficiente para !ue ese estado de cosas sea posible lgicamente. b2 convencerse de !ue es realmente posible !ue existamos sin el cuerpo" o sea" !ue es claro y distinto !ue puedo existir sin un cuerpo. c2 imaginarse !ue uno es tal y cual cosa lo lleva a sostener !ue lo !ue es imaginable 4concebible2 es verdad. &) El arg)mento de la Di4isi/ilidad, tam/i8n presentado por Descartes tiende a pro/ar ()e 5o no p)edo ser lo mismo ()e mi c)erpo p)esto ()e: a2 yo" como sujeto de experiencia" me manifiesto a m mismo como con diversos estados de conciencia 4divisibilidad2" mientras !ue mi cuerpo" al no poseer conciencia" debe ser un *nico cuerpo 4unicidad o simplicidad2. b2 yo" como sujeto de experiencias" soy simple e indivisible" mientras !ue mi cuerpo obviamente es divisible" por ser extenso. 0) En realidad, si mi c)erpo es extenso, como toda materia $para Descartes), todo lo ()e es extenso tiene partes, 5 por lo tanto es di4isi/le, 5 no simplea2 una persona se concibe como poseedora de propiedades mentales y de un cuerpo" del cual hace uso o habita en l. b2 una persona se concibe como poseedora de propiedades fsicas !ue permiten la emergencia de estados mentales" a diferencia de otros cuerpos fsicos !ue no manifiestan esas emergencias 4los animales no humanos" por ejemplo2. c2 una persona se concibe como poseyendo caractersticas fsicas y mentales: siente" piensa" pero tambin ocupa un lugar y tiene una masa.

2) <)ando !oKe define as a la persona, pretende escapar a las crticas ()e reci/i el D)alismo <artesiano- De las sig)ientes opciones marca a()ella !ue efectivamente no se corresponde con la nocin d)alista de !oKeEl s)6eto de experiencias es )na cosa ()e posee caractersticas tanto fsicas como mentalesa2 La persona no es idntica con el cuerpo b2 La persona es diferente de un alma inmaterial c2 +e afirma una identidad cuerpo3persona d2 La persona no se concibe como una combinacin de cuerpo y alma. >) Ba5 alg)nas o/6eciones concept)ales a la interaccin d)alista, de la c)al !oKe tendr. ()e defenderse, para mantener s) posicin d)alistaDeale c).l no es )na o/6ecin concept)al, tal 5 como las presenta !oKea2 La nocin de relacin causal" !ue pide !ue tanto la causa como el efecto compartan un lugar" fue abandonada hace ;<< a0os" a partir de la ley de gravitacin universal de -e1ton. b2 no tiene sentido decir !ue hay 9transaccin causal9 entre substancias tan radicalmente diferentes. c2 La transaccin causal es concebida como operando localmente 4no accin a distancia" y menos entre algo espacial3fsico y algo !ue no tiene ubicacin espacial2. d2 La transaccin causal implica transmisin de alguna propiedad de la causa al efecto. = en el dualismo cartesiano las propiedades de causa y efecto son totalmente diversas2. ?) Ieamos a7ora las o/6eciones empricas a la interaccin d)alista- En ellas se plantea ()e, en la interaccin d)alista no se conser4ara el momento $o sea, la cantidad de mo4imiento en el m)ndo), p)esto ()e: a2 la interaccin causal mente3cuerpo no implica acrecentar la cantidad de movimiento en el mundo" ni en 'escartes ni en -e1ton b2 la mente" como realidad no3fsica" si fuese causa de alg*n movimiento" agregara m s movimiento al universo" lo cual es inconcebible en la mec nica de 'escartes y m s tarde en la de -e1ton tambin. c2 puesto !ue la mente no interact*a causalmente con el cuerpo de manera directa sino a travs de la gl ndula pineal 1A) <)ando !oKe niega ()e el c)erpo sea id8ntico a la persona apela a ()e la persona posee distintas condiciones de persistencia , adem.s de apo5arse en ()e, c)ando )no piensa o siente, no se lo atri/)5e a su cuerpo o a su cerebro, sino a s mismo- En arg)mento ()e a5)da a comprender esto es a()el de la Analoga de la estatua de bronce y del tro!o de bronce- Del mismo modo ()e la estat)a no p)ede ser id8ntica al tro"o de /ronce, la persona no p)ede ser id8ntica a su cuerpo- Adem.s de ()e la estat)a m)5 pro/a/lemente 7a5a comen"ado a existir despu"s del tro"o de ()e est. 7ec7a, tam/i8n es pro/a/le ()e de6e de existir antes ()e el tro"o de /ronce- $Aplaste )sted la estat)a, 5 5a no la tendr. m.s, a)n()e conser4ar. el tro"o de /ronce-) El arg)mento ser4ir. para explicar ()e persona 5 su cuerpo son an.logos en este sentido- Deale c).l de las sig)ientes opciones no se sigue de esta analogaa2 )anto la estatua como el trozo de bronce tienen caractersticas fsicas b2 La persona" como la estatua" tiene la misma forma y masa !ue su cuerpo" sin ser idntica a ese cuerpo c2 ,l igual !ue la estatua se compone del trozo de bronce" la persona se compone por su cuerpo

<!AEDERA <AEDA! DE !' +ODI<' $!'NE)


1) Repasamos a7ora la presentacin del Arg)mento de <la)s)ra <a)sal%ara ()e Ested se )/i()e, primero aseg1rese ()e entiende de dnde pro4iene este arg)mento: a2 Es un argumento a favor del 'ualismo +ubstancial -o3Cartesiano" tal y como lo sostiene Lo1e 4) Es un argumento del fisicismo en contra de la posicin 'ualista $nteraccionista en general. 5) Es un argumento del fisicismo en contra del 'ualismo +ubstancial -o3Cartesiano de Lo1e. ) En el arg)mento de <la)s)ra <a)sal 7a5 # premisas 5 )na concl)sin fisicalista- !o importante es ()e Ested $imagnese ()e es )n d)alista) de/e aceptar las # premisas- %ero, si 7ace esto, 5 no ()iere caer en )na contradiccin, tendr. ()e con4ertirse en )n fisicalista- ' sea, de/er. aceptar la concl)sin- !a primera premisa dice ()e en c)al()ier momento ()e )n estado fsico tiene )na ca)sa, tiene )na ca)sa fsica completamente s)ficiente- Esta premisa es la llamada %rincipio de <la)s)ra <a)sal de lo +sico, 5 da nom/re al arg)mento- ;M)8 se est. diciendo en esta premisa= a2 +e dice !ue en un momento cual!uiera previo a un efecto debe haber una causa fsica !ue baste para !ue el efecto ocurra" pero no es necesario determinar cu l es ese momento cual!uiera. b2 +e dice !ue en cada instante previo a un efecto fsico hay una causa fsica !ue basta para !ue ese efecto se d. > sea" hay tantas causas fsicas" conectadas linealmente" como instantes !ue anteceden al efecto c2 +e dice !ue" no importa el momento previo !ue usted escoja antes de un estado fsico" siempre hallar una causa suficiente. #) !a seg)nda premisa del Arg)mento de <la)s)ra <a)sal dice ()e entre las ca)sas de alg)nos estados fsicos se enc)entran estados mentalesa2 Este es el llamado &rincipio de Causacin &sico3.sica" y obviamente es lo !ue sostiene el dualista interaccionista b2 Este es el llamado &rincipio de Causacin &sico3.sica" aun!ue" si somos dualistas" no podramos aceptarlo" pues entra en contradiccin con la premisa uno !ue ya aceptamos &) !a tercera premisa del Arg)mento de <la)s)ra <a)sal dice ()e, c)ando )n estado fsico tiene )n estado mental entre s)s ca)sas, raramente s)cede, si es ()e s)cede alg)na 4e", ()e este estado fsico est8 so/redeterminado por ese estado mental 5 otro estado fsico- Este %rincipio de no so/redeterminacin ca)sal lo ()e ()iere esta/lecer es ()e: a2 un estado fsico cual!uiera" por ejemplo" no puede estar sucesivamente determinado por dos causas anteriores encadenadas en el tiempo" y adem s esas dos causas anteriores ser enteramente suficientes b2 un efecto fsico" por ejemplo" una conducta suya de acercarse a una manzana" no puede estar suficientemente determinado por un estado fsico 4por ejemplo" cerebral2 y a la vez 4al mismo tiempo2 y adem s suficientemente" por un estado mental 4por ejemplo una creencia de !ue comer manzanas ayuda a mantener mi peso2. c2 (n evento fsico no se halla excesivamente determinado por un estado mental y uno fsico" puesto !ue uno slo de ellos bastara para causarlo" y apelar a los dos para su explicacin o causacin sera como sobre3determinarlo 0) !a concl)sin del Arg)mento de <la)s)ra <a)sal dice ()e al menos alg)nos estados mentales son id8nticos a ciertos estados fsicos- Esto se sig)e de las tres premisas p)esto ()e: a2 +i la interaccin psico3fsica de la premisa ? sobredetermina al efecto fsico 4por ejemplo" a una conducta2" entonces no podramos decidir si la causa de la

conducta es lo mental o lo fsico. &or ello debemos admitir por ? y ; !ue slo hay causas fsicas anteriores al evento" y esto es lo !ue dice la premisa @. b2 +i la cadena causal fsica anterior al evento est densamente poblada de causas fsicas suficientes" no hay lugar para poner otra causa mental" a no ser !ue esta sea tambin fsica 4!ue es la conclusin !ue se sigue2. = puesto !ue no hay sobre3determinacin causal 4o sea" causas paralelas independientes y suficientes para tal evento2" debemos aceptar el fisicismo. 2) !oKe o/6etar. el Arg)mento de <la)s)ra <a)sal, atacando la premisa 1 $el %rincipio de <la)s)ra <a)sal)- ;%or ()8 no atacar. a la o a la #= a2 &or!ue al atacar la @ basta para admitir el dualismo. $ncluso si podemos criticar la @" no hara falta afirmar la ? y ; para !ue seamos dualistas. b2 -o atacar a la ? y a la ;" por!ue la ? es la premisa del dualista" y no podemos excluirla del argumento" si !ueremos convencer al dualista de algo. La premisa ; es algo !ue damos por sentado >) !a modificacin ()e !oKe 7ar. de la premisa 1 gira en torno a esa cadena ca)sal fsica cla)s)rada a ()e al)de- !ea la premisa 1G 5 seale c).l de las interpretaciones ()e sig)en es la adec)ada para esa 1Ga2 en @A se dice !ue para cada evento fsico" por ejemplo" una conducta" hay una causa fsica" pero no en todo momento" sino slo en alguno o algunos. 'e este modo" podra haber tambin una causa mental luego de la fsica y antes del efecto b2 en @A se dice !ue para cada evento fsico nuestro 4una conducta2" slo existe una causa fsica" y el resto de ellas son causas mentales 4creencias" deseos" sensaciones" etc.2 ?) En 1ltimo arg)mento contra el interaccionismo ()e presenta !oKe es el del Fonismo Anmalo, de Donald Da4idson- !ee las # premisas 5 la concl)sin del arg)mento, 5 seala c).l de las sig)ientes interpretaciones es la correctaa2 +i habl ramos de interaccin causal entre lo !ue sea 4mente3cuerpo" cuerpo3 cuerpo" mente3mente2 deberamos someter esa interaccin causal a una ley determinista. > sea" toda relacin causal es determinista estricta. &ero si nos emperramos en sostener !ue la mente interact*a causalmente con el cuerpo" entonces esa interaccin tendra !ue ser determinista estricta. +in embargo" no hay leyes deterministas estrictas para la interaccin de nuestros estados mentales con los corporales. b2 Bablar de interaccin causal mente3cuerpo es hablar de leyes deterministas estrictas. > sea" un estado mental 4la creencia de !ue el fuego !uema" por ejemplo2 s o s determina !ue llevemos a cabo una conducta determinada 4la conducta de alejar la mano del fuego2. = como siempre la creencia nos determina una conducta" entonces" la creencia 4la causa2 debe ser fsica" puesto !ue la conducta 4el efecto2 es fsica" y las interacciones causales slo son entre eventos fsicos. @) '/6eciones al arg)mento del Fonismo Anmalo- !oKe presenta 4arias o/6eciones a este arg)mento, ()e son las sig)ientes, excepto )na de ellas ()e no lo es- ;<).l sera= a2 Las interacciones causales no siempre caen bajo una ley causal general b2 Es posible pensar un tipo de interaccin entre estados mentales y estados fsicos !ue no re!uiera !ue sean idnticos entre s. c2 Bay leyes fsicas !ue hablan de interaccin y !ue no son deterministas estrictas" o sea" !ue admiten excepciones. d2 Los acaecimientos mentales no interaccionan causalmente con acaecimientos fsicos.

<'NDE<PIDF'
1 El <ond)ctismo +ilosfico $o lgico) 5 el %sicolgico a. )ienen la misma base epistemolgica b. +on conceptualmente iguales pero metodolgicamente diferentes c. )ienen distinta base epistemolgica d. +on conceptualmente diferentes pero coinciden histricamente El cond)ctismo s)rge por: a. )odas son correctas b. $nfluencia del positivismo lgico y por considerar a los problemas filosficos como confusiones lingCsticas c. Considerar !ue la psicologa deba tener un firme contacto con la realidad emprica 4reaccin al 'ualismo2 # El pro/lema Fente 9 <)erpo seg1n el <ond)ctismo a. -o tiene solucin b. -inguna es correcta c. Es de inters central d. Es un pseudo problema & !a critica reali"ada al cond)ctismo acerca de la lista de disposiciones de m1ltiples 4as, se f)ndamenta en ()e: a. Es un legado de la ciencia positivista b. Carece de sentido -o es real c. Es imposible poder aprenderla 0 !os Estados Fentales para )n <ond)ctista !gico $o +ilosfico) a. +on consecuencia de la conducta b. +on disposiciones de m*ltiples vas c. +on definiciones operacionales d. -o existen 2 DQinner present )na 4ersin reciente del <ond)ctismo donde a. Los estados !ue discriminamos en la introspeccin son causas reales de la conducta b. , travs de la introspeccin vemos nuestros estados mentales c. Considera a los fenmenos internos y el acceso introspectivo a ellos d. , y C son correctas

<'NDE<PIDF'
1 !os cond)ctistas ()e sostienen ()e el 1nico tipo de datos ()e podemos tener relati4os a los estados mentales de c)al()iera, incl)5endo los n)estros se enc)entran en la cond)cta externamente o/ser4a/le, tanto 4er/al como no 4er/al se denominan: a. conductistas lgicos b. conductistas cientficos c. conductistas sem nticos d. conductistas filosficos R5le sostiene ()e las pala/ras disposicionales p)eden ser altamente gen8ricas o altamente especficas- De ac)erdo a esto RgenerosoR 5 R/e/edor de coca colaR pertenecen a a. no pertenecen a la misma categora de palabras disposicionales b. 9generoso9 es una palabra altamente especfica c. la misma categora de palabras disposicionales

d. 9bebedor de coca cola9 es una palabra altamente genrica # Dentro del sistema cartesiano: a. (na de las marcas de lo mental es la racionalidad b. (na de las marcas de lo mental es la privacidad c. (na de las marcas de lo mental es la intencionalidad d. (na de las marcas de lo mental es la emocionalidad & Deg1n !oKe, el f)ncionalismo no p)ede explicar los estados mentales por()e s) teori"acin: a. no es compatible con un fisicismo no3reductivo b. no tiene en cuenta la existencia de la conciencia. c. no es realista en su concepcin de los estados mentales y la intencionalidad 0 Eno de los principales s)p)esto de las ideas de Descartes es ()e: a. todas las ideas cruciales en el entendimiento humano se ad!uiran por medio de la experiencia b. algunas de las ideas cruciales en el entendimiento humano son innatas c. no es posible rastrear el origen de las ideas cruciales !ue componen al entendimiento humano

<'NDE<PIDF' #
En la actualidad" pueden identificarse 4al menos2 dos clases de conductismo. &ese a !ue" en ocasiones" son identificados bajo distintas denominaciones" esta clasificacin es compartida por los autores de la bibliografa obligatoria: a2 conductismo filosfico o lgico" b2 conductismo metodolgico o cientfico. 1) El cond)ctismo filosfico o lgico: a2 Es una posicin filosfica !ue sostiene !ue: aun!ue los fenmenos mentales existen" en tanto estados internos !ue causan la conducta" stos deben ser definidos en trminos de disposiciones a la conducta para lograr mayor claridad en la comprensin cientfica de estos fenmenos. b2 Es una posicin filosfica !ue propone definir los trminos mentales ordinarios mediante trminos conductuales. &ara ello" utiliza definiciones operacionales como las empleadas para definir propiedades disposicionales. ) !os cond)ctistas filosficos comparten el sig)iente s)p)esto: a2 Los problemas filosficos son" por lo general" problemas ontolgicos" debido a !ue tratan sobre la existencia de cierto tipo de entidades" tales como el alma o la mente. b2 Los problemas filosficos surgen" en su mayora" debido a confusiones lingCsticas o sem nticas" !ue pueden resolverse analizando los usos lingCsticos o el vocabulario. c2 Los problemas filosficos son problemas metodolgicos !ue desaparecer n cuando las distintas ramas de la filosofa se conviertan en disciplinas cientficas. #) %ara !oKe, el cond)ctismo epistemolgico p)ede ser caracteri"ado del sig)iente modo: a2 >frece primero una explicacin de !u son los estados mentales" para luego caracterizar cmo debemos conocer los estados mentales. b2 >frece una explicacin de !u son los estados mentales" aun!ue no explica cmo debemos conocerlos. c2 >frece una caracterizacin de cmo debemos conocer los estados mentales" aun!ue no explica !u son los estados mentales.

PE'ROAD DE !A IDENPIDAD S $DEAR!E)


1) !os tericos de identidad defendan ()e el 7ec7o de ()e la mente f)era id8ntica al c)erpo era )na 4erdad: a2 !ue deba ser descubierta por las ciencias empricas b2 !ue deba ser postulada por la ciencia emprica 4como la fsica2 c2 !ue deba ser descubierta por las ciencias formales 4como la matem tica2 ) %ara los tericos de la identidad el d)alismo s)stancial era: a2 una posibilidad terica. b2 una cuestin !ue deba ser negada por la ciencia desde sus principios c2 un absurdo !ue implicaba una contradiccin #) !o ()e afirma la teora de la identidad es ()e los fenmenos mentales son: a2 reducibles casi por completo a lo fsico b2 en gran medida fenmenos fsicos c2 nada m s !ue algo fsico &) Di el ()e los fenmenos mentales sean id8nticos a lo fsico depende de )na in4estigacin emprica entonces: a2 un terico de la identidad debera necesariamente y a priori aceptar !ue los fenmenos mentales son fsicos b2 un terico de la identidad no podra nunca aceptar" bajo ninguna condicin lgica" !ue los fenmenos mentales se reducen a lo fsico c2 un terico de la identidad" bajo ciertas condiciones y en alg*n mundo posible" podra aceptar !ue los fenmenos mentales no se reducen a lo fsico 0) De ac)erdo con TripQe, el terico de la identidad enfrenta el sig)iente pro/lema: Di es 4erdad ()e 7a5 )na identidad entre estado mental 5 estado fsico: a2 esto debera ser una verdad emprica. b2 esto debera ser una verdad necesaria c2 esto debera ser una verdad altamente probable 2) Panto los cond)ctistas lgicos, como los tericos de la identidad 7an afirmado )na tesis ontolgica en relacin con los estados mentales- %ero, de ac)erdo con Dearle, )na diferencia entre am/as post)ras consistira en ()e: a2 /ientras los conductistas afirmaban !ue la naturaleza de los estados mentales era una consecuencia de los avances empricos de la ciencia" los tericos de la identidad afirmaban !ue este era un problema de definiciones. b2 /ientras !ue los conductistas lgicos afirmaban !ue la naturaleza de los estados mentales era una consecuencia de la forma en cual eran definidos" los tericos de la identidad afirmaban !ue esta era una cuestin f ctica. c2 /ientras los conductistas lgicos afirmaban !ue la naturaleza de los estados mentales era una consecuencia de los avances empricos de la ciencia" los tericos de la identidad afirmaban !ue era esta era una cuestin f ctica. >) Entre las o/6eciones ()e se presentaron al terico de la identidad, 7a5 )na ()e Dearle considera como con4incente- De ac)erdo con el terico de la identidad, de/o ser capa" de esta/lecer )na identidad entre dos fenmenos, lo c)al in4ol)cra, ()e de/o esta/lecer ()e 7a5 )na misma cosa a partir de dos gr)pos de propiedades- En t8rminos del terico de la identidad est.ndar esto significa ()e el en)nciado 3el dolor es id8ntico a cierto estado cere/ral: s)pone ()e: a2 las propiedades de los estados mentales no pueden reducirse a propiedades meramente fsicas b2 las propiedades de los estados mentales deben poder reducirse a propiedades meramente fsicas c2 las propiedades de los estados mentales pueden reducirse en gran medida a propiedades meramente fsicas

?) )na o/6ecin a la teora de la identidad f)e el llamado 3c7o4inismo ne)ronal:- Esta o/6ecin f)e )n pro/lema importante para la teora de identidad tipo9tipo- D)pongamos ()e de ac)erdo con la teora de la identidad tipo9tipo, se dice ()e 3todo dolor es id8ntico a )n estado fsico:Esta afirmacin excl)5e: $4arias opciones posi/les) a2 7ue haya dolores !ue no tengan como base un estado fsico b2 7ue haya dolores !ue tengan como base otra clase de estado fsico c2 7ue haya dolores !ue tengan como base otro estado fsico particular

FAPERIA!IDF' $DEAR!E)
1) Deg1n el materialismo eliminati4o, la in4estigacin en ne)rociencias no permitira 7allar correlaciones entre el 4oca/)lario mentalista de la %sicologa %op)lar 5 el 4oca/)lario de las Ne)rociencias- El f)ndamento es ()e la red)ccin entre am/as teoras es imposi/leDerdadero .also

) Eli6a la me6or descripcin de la sig)iente proposicin: RPodo estado mental p)ede reali"arse en m.s de )n estado fsicoR,. conductismos E. materialismo eliminativo C. dualismo sustancia '. paralelismo psicofsico E. funcionalismos #) De ac)erdo con la teora de la identidad, los e4entos mentales son e4entos electro()micos en el cere/ro- Desde tal perspecti4a, decir ()e )na persona tiene )na mente, significa ()e: ,. la persona muestra una conducta !ue indica !ue la persona est viva E. la mente de la persona causa un patrn de actividad neural complejo y mensurable C. la persona tiene un cerebro y un sistema nervioso complejo '. la activacin neural en el cerebro demuestra la presencia de una conciencia espiritual &) El Fonismo Anmalo es: ,. un dualismo de propiedades E. un materialismo eliminativo C. una versin del funcionalismo '. un forma de conductismo E. una teora de la identidad 0) De/e artic)lar cada teora con la o/6ecin de sentido com1n ()e se le reali"aH /ase s)s resp)estas en los textos de Dearle@. )eora de la identidad de instancias ?. .uncionalismo de m !uina 4$, .2 ;. F. H. I. Conductismo lgico .uncionalismo de la caja negra /aterialismo eliminativo )eora de la identidad de tipos a. Leva el dualismo de propiedades. b. -o da cuenta de la mente" seg*n el argumento de la habitacin china c. -o considera los !ualia ausentes d. +uperactor G +uperespartano e. -iega la existencia de la mente f. -o considera los !ualia ausente ni el espectro invertido

2) De ac)erdo con +lanagan, la post)ra de +re)d respecto al pro/lema mente9cere/ro es interaccionista, en tanto descri/e enfermedades psicosom.ticas mostrando ()e cere/ro es la /ase de la mente 5 la mente p)ede infl)ir en el cere/ro9c)erpoDerdadero .also

>) De ac)erdo con <7)rc7land, )na de las metas principales del f)ncionalismo de la psicologa pop)lar es /)scar cmo entender los t8rminos mentales apelando al cond)ctismo filosfico 5 a las ne)rocienciasDerdadero .also

?) El arg)mento de ()e mis estados mentales son conocidos introspecti4amente por m, mientras ()e mis estados cere/rales no lo son, 5 por lo tanto mis estados mentales no p)eden ser id8nticos a mis estados cere/rales, es )n arg)mento ()e trata de ref)tar a la teora de la identidad como tipo, pero no alcan"a a la teora de la identidad como instanciaDerdadero .also

@) Deale si la sig)iente afirmacin es 4erdadera o falsa: REn el ordenador 7)mano de P)ring 7a5 realmente )n ni4el de programa intrnseco al sistema 5 est. f)ncionando ca)salmente a ese ni4el para con4ertir inp)t en o)tp)tRDerdadero .also

1A) Deg1n Dearle, las distintas 4ersiones del materialismo excl)5en los mismos rasgos de 4ida mental: estados concientes e intencionalesDerdadero .also

11) !a tesis de la identidad mente9c)erpo es ,. (na verdad a priori acerca de la naturaleza fsica de los estados mentales E. (na afirmacin ontolgica" f ctica acerca de la naturaleza fsica de los estados mentales C. (na afirmacin sem ntica acerca del significado del vocabulario mental 1 ) En las teoras de la identidad mente9cere/ro, la distincin RtipoR e RinstanciaR se refiere a la diferencia entre e4entos mentales 5 clases ne)rofisiolgicasDerdadero .also

+EN<I'NA!IDF' $<BER<B!AND)
1) Deg1n el f)ncionalismo el rasgo esencial de todo tipo de estado mental es: a2 el conjunto de propiedades fsicas por medio de las cu les podemos identificarlo en relacin al entorno" otros estados mentales y la conducta del organismo. b2 el conjunto de propiedades estrictamente mentales" entre las cuales la m s importante es la de permitir el pensamiento. c2 el conjunto de relaciones causales con estmulos del entorno" otros estados mentales y la conducta del organismo. ) Ena diferencia m)5 importante entre el f)ncionalismo 5 el cond)ctismo filosfico es ()e, el cond)ctismo niega: a2 !ue los estados mentales existan. b2 !ue los estados mentales puedan caracterizarse identificando sus propiedades no fsicas c2 la plausibilidad de la teora de la identidad. #) !a diferencia entre el f)ncionalismo 5 la teora de la identidad es ()e, el f)ncionalismo implica ()e: a2 un tipo de estado mental podra ser idntico a estados fsicos de cual!uier tipo. b2 un tipo de estado mental es idntico a un tipo de estado no fsico c2 un tipo de estado mental es idntico a un tipo de estado fsico. &) El f)ncionalismo 5 la teora de la identidad de instancias son: a2 ,mbas incompatibles con el materialismo b2 $ncompatibles entre s. c2 Compatibles entre s. 0) El f)ncionalismo es compati/le con: a2 ambos: el materialismo y el dualismo de sustancias b2 el materialismo. c2 el dualismo de sustancias. 2) !o ()e se conoce como 3arg)mento del espectro in4ertido:: a2 ,taca la tesis del funcionalismo. b2 ,poya la tesis del funcionalismo c2 Es irrelevante para la tesis del funcionalismo. >) En arg)mento en contra de la tesis del f)ncionalismo se conoce como: a2 ,rgumento de la dependencia nerviosa. b2 ,rgumento de los !ualia ausente. c2 ,rgumento de la divisibilidad.

FAPERIA!IDF' E!IFINAPII' $<BER<B!AND)


1) El materialista eliminati4o defiende: a2 (na perspectiva ontolgica del problema mente 3 cuerpo b2 (na perspectiva estrictamente metodolgica del problema mente3 cuerpo c2 (na perspectiva acorde con la elaborada por el funcionalista en relacin con el problema mente3 cuerpo. ) El materialista eliminati4o tiene )na posicin f)erte respecto a la existencia de estados mentales: a2 Considera !ue en gran medida han sido probados por medio de estudios neurofisiolgicos. b2 Cree !ue los estados mentales tienen !ue poder reducirse a estados fsicos si han de ser aceptados por la psicologa cientfica. c2 +ostiene !ue los estados mentales no existen. #) El materialista eliminati4o discrepa con el materialista de la identidad de tipos: a2 &or!ue considera !ue nunca se avanzar lo suficiente en relacin con el estudio del sustrato fsico como para llegar a establecer las relaciones biunvocas. b2 &or!ue cree !ue hay m s de un estado fsico !ue podra ejemplificar un determinado estado mental. c2 &or!ue considera !ue no ser nunca posible lograr la reduccin interterica" debido a la imprecisin de la teora psicolgica popular. &) El materialista eliminati4o cree ()e el 4oca/)lario: a2 7ue utilizamos se perfeccionar conforme se desarrollen las teoras psicolgicas introspectivas. b2 7ue el vocabulario se modificar radicalmente cuando se conozcan los estados cerebrales a los !ue referamos imprecisamente mediante el vocabulario mentalista. c2 7ue el vocabulario cientfico se modificar parcialmente cuando se conozca suficientemente el sustrato fisiolgico" por!ue ya no se har referencia a algunos Jestados mentalesK. 0) !os arg)mentos ()e c)entan a fa4or del materialista eliminati4o son: a2 El de la incoherencia interna" de la insuficiencia explicativa y de los !ualia ausentes. b2 $nsuficiencia explicativa" leccin inductiva de nuestra historia conceptual" ventaja a priori. c2 $nsuficiencia explicativa" ventaja a priori" argumento del espectro invertido. 2) !os arg)mentos ()e se )tili"an en contra del materialista eliminati4o son: a2 El argumento de la introspeccin y de los !ualia ausentes b2 El de la navaja de >ccam" de la historia evolutiva y de la introspeccin. c2 El de la introspeccin" la incoherencia interna y el de la concepcin alarmista.

PEDID DE !A AEP'N'FOA $+!ANAGAN)


>) !a posicin inicial de +re)d: a2 +e contrapona a la neurociencia. b2 &ropiciaba el estudio de la mente basado en la investigacin neurocientfica. c2 &romova una teora de la identidad de instancias. ?) En seg)nda instancia +re)d 7a/ra pasado a sostener )na posicin: a2 /aterialista laxa" cuanto m s de identidad de instancia. b2 'ualista de propiedades c2 Eliminativista" tendiente a mostrar los procesos ps!uicos como estados cuantitativamente determinados. @) En tercera instancia +re)d defendera la a)tonoma de la psicologa /as.ndose en: a2 7ue el vocabulario neurofisiolgico podra ofrecer explicaciones completas" pero resulta muy complicado de usar en una terapia. b2 7ue los estudios de neurofisiologa no ofrecen m s !ue una mascarada in*til" incapaz de explicar los procesos psicolgicos. c2 7ue los estudios neurofisiolgicos no constituyen el eje de la reflexin psicloga" pero suman al momento de comprender las emisiones de los pacientes. 1A) +lanagan considera ()e +re)d, ante el pro/lema ontolgico del pro/lema mente9c)erpo defiende: a2 &as a defender un dualismo fuerte" por!ue consider !ue la intencionalidad es una propiedad exclusiva de lo mental. b2 -o asumi una posicin" pero esta sera m s cercana al dualismo de propiedades. c2 Continu siendo materialista" pero cuestion la posibilidad de !ue el vocabulario de las neurociencias pudiera expresar la totalidad de la vida ps!uica.

%R'U!EFA 'NP'!VGI<'
1) Eli6a la me6or descripcin de la sig)iente proposicin: RPodo estado mental p)ede reali"arse en m.s de )n estado fsicoRa. materialismo eliminativo b. funcionalismos c. conductismos d. dualismo sustancia e. paralelismo psicofsico ) El Fonismo Anmalo es: a. un materialismo eliminativo b. un dualismo de propiedades c. una versin del funcionalismo d. una teora de la identidad e. un forma de conductismo #) De/e artic)lar cada teora con la o/6ecin de sentido com1n ()e se le reali"aH /ase s)s resp)estas en los textos de Dearle- $Enir con flec7as) a. /aterialismo eliminativo @. Lleva al dualismo de propiedades b. )eora de la identidad de tipos ?. >bjeciones del superactorGsuperespartano c. .uncionalismo de la caja negra ;. -o considera los !ualia ausentes d. )eora de la identidad de instancias F. -o considera los !ualia ausentes y el esp. inv. e. Conductismo lgico H. -o da cuenta de la mente 4seg*n Bab. China2 f. .uncionalismo de m !uina 4$, fuerte2 I. -iega la existencia de la mente 4injusto con &&2 &) En las teoras de la identidad mente9cere/ro, la distincin RtipoR e RinstanciaR se refiere a la diferencia entre e4entos mentales 5 clases ne)rofisiolgicasDerdadero .also 0) Deg1n Dearle, las distintas 4ersiones del materialismo excl)5en los mismos rasgos de 4ida mental: estados concientes e intencionalesDerdadero .also 2) El arg)mento de ()e mis estados mentales son conocidos introspecti4amente por m, mientras ()e mis estados cere/rales no lo son, 5 por lo tanto mis estados mentales no p)eden ser id8nticos a mis estados cere/rales, es )n arg)mento ()e trata de ref)tar a la teora de la identidad como tipo, pero no alcan"a a la teora de la identidad como instanciaDerdadero .also >) De ac)erdo con la teora de la identidad, los e4entos mentales son e4entos electro()micos en el cere/ro- Desde tal perspecti4a, decir ()e )na persona tiene )na mente, significa ()e: a. la mente de la persona causa un patrn de actividad neural complejo y mensurable b. la activacin neural en el cerebro demuestra la presencia de una conciencia espiritual c. la persona muestra una conducta !ue indica !ue la persona est viva d. la persona tiene un cerebro y un sistema nervioso complejo ?) !a tesis de la identidad mente9c)erpo es: a. (na afirmacin sem ntica acerca del significado del vocabulario mental b. (na afirmacin ontolgica" f ctica acerca de la naturaleza fsica de los estados mentales c. (na verdad a priori acerca de la naturaleza fsica de los estados mentales

@) De ac)erdo con <7)rc7land, )na de las metas principales del f)ncionalismo de la psicologa pop)lar es /)scar cmo entender los t8rminos mentales apelando al cond)ctismo filosfico 5 a las ne)rocienciasDerdadero .also 1A) Deale si la sig)iente afirmacin es 4erdadera o falsa: REn el ordenador 7)mano de P)ring 7a5 realmente )n ni4el de programa intrnseco al sistema 5 est. f)ncionando ca)salmente a ese ni4el para con4ertir inp)t en o)tp)tRDerdadero .also 11) De ac)erdo con +lanagan, la post)ra de +re)d respecto al pro/lema mente9cere/ro es interaccionista, en tanto descri/e enfermedades psicosom.ticas mostrando ()e cere/ro es la /ase de la mente 5 la mente p)ede infl)ir en el cere/ro9c)erpoDerdadero .also 1 ) Deg1n el materialismo eliminati4o, la in4estigacin en ne)rociencias no permitira 7allar correlaciones entre el 4oca/)lario mentalista de la %sicologa %op)lar 5 el 4oca/)lario de las Ne)rociencias- El f)ndamento es ()e la red)ccin entre am/as teoras es imposi/leDerdadero .also

%R'U!EFA 'NP'!VGI<'
1) !ea la sig)iente afirmacin e indi()e a ()8 tipo de cond)ctismo pertenece: R!as explicaciones en psicologa de/en /asarse totalmente en nociones ()e sean directamente o/ser4a/les por todos o definidas operacionalmente en trminos de conceptos as observables9. a. pertenece al conductismo filosfico b. pertenece al conductismo lgico c. pertenece al conductismo metodolgico ) El d)alismo de propiedades es, en sentido general, )na prop)esta: a. reduccionista b. no reduccionista c. indiferente al problema del reduccionismo #) <7)rc7land afirma ()e la diferencia entre el epifenomenismo 5 el d)alismo interaccionista de propiedades es ()e: a. el epifenomenismo sostiene !ue las propiedades mentales tienen efectos casuales y el dualismo de propiedades dice !ue no b. el epifenomenismo sostiene !ue las propiedades mentales tienen solo efectos causales parciales" mientras !ue el dualismo interaccionista dice !ue no c. el epifenomenismo sostiene !ue las propiedades mentales no tienen efectos causales" mientras !ue el dualismo interaccionista de propiedades sostiene !ue si &) El d)alismo de s)/stancia cartesiano es )n d)alismo interaccionistaEsto ()iere decir ()e: a. los estados mentales pueden causar estados fsicos" pero los estados fsicos no pueden causar estados mentales b. los estados mentales no pueden causar estados fsicos" pero los estados fsicos pueden causar estados mentales

c. los estados mentales pueden causar estados fsicos y los estados fsicos pueden causar estados mentales d. los estados mentales no pueden causar estados fsicos y los estados fsicos no pueden causar estados mentales 0) Deg1n Dearle, el cond)ctismo lgico se 7a enfrentado a dos tipos de o/6eciones: (Seleccione al menos una respuesta) a. las objeciones de ndole tcnica b. las objeciones neurocientficas c. las objeciones de car cter funcionalista d. las objeciones de sentido com*n 2) Deg1n Dearle, el cond)ctismo metodolgico se de/ilit 5 f)e rec7a"ado por()e a. confundi el 9tema9 de estudio de la psicologa 4la mente2 con 9las pruebas9 de la existencia de dicho tema 4el comportamiento humano2 b. los argumentos conductistas solo pudieron dar cuenta de las relaciones causales existente entre los estados mentales y los comportamientos c. solo el conductismo filosfico pudo proponer una explicacin plausible de cmo podan traducirse proposiciones sobre la mente en proposiciones sobre el comportamiento >) Deg1n <7)rc7land, a)n()e el d)alismo no es la concepcin m.s dif)ndida dentro de la com)nidad filosfica 5 cientfica act)al, es sin em/argo la teora m.s pop)lari"ada so/re la mente- En opinin de este a)tor, esto se de/era a: a. la influencia de las religiones universales b. la influencia de la ciencia c. la influencia de la filosofa ?) Deg1n Dearle, los d)alistas act)ales se enfrentan a )n pro/lema adicional: la le5 fsica de la conser4acin de la materia, ()e dice ()e la cantidad de materiaWenerga en el )ni4erso es constante- Di el d)alismo es 4erdadero, a. debera abandonarse la concepcin cl sica de la relacin materiaGenerga y adoptar una teora cu ntica probabilstica b. las explicaciones sobre el funcionamiento neurofisiolgico del cerebro y su estructura anatmica" seran falsas c. una ley fundamental de la fsica" como es la conservacin de la materia" sera falsa @) El principio de la cla)s)ra ca)sal de lo fsico, ()e forma parte del arg)mento de la cla)s)ra ca)sal, sostiene ()e: a. en cual!uier momento !ue un estado fsico tenga una causa" tiene una causa fsica enteramente suficiente b. siempre !ue un estado fsico tenga una causa" tiene una causa no fsica enteramente suficiente 1A) De ac)erdo con Descartes, a las m.()inas n)nca les ser. posi/le (Seleccione al menos una respuesta) a. )ener la misma apariencia !ue nosotros b. Bacer matem tica como nosotros c. Bablar como nosotros d. Caminar como nosotros

%R'U!EFA 'NP'!VGI<' #
1) El d)alismo de s)/stancia cartesiano es )n d)alismo interaccionistaEsto ()iere decir ()e:

,. los estados mentales no pueden causar estados fsicos y los estados fsicos no pueden causar estados mentales $ncorrecto E. los estados mentales no pueden causar estados fsicos" pero los estados fsicos pueden causar estados mentales $ncorrecto C. los estados mentales pueden causar estados fsicos y los estados fsicos pueden causar estados mentales '. los estados mentales pueden causar estados fsicos" pero los estados fsicos no pueden causar estados mentales $ncorrecto ) De ac)erdo con Descartes, a las m.()inas n)nca les ser. posi/le +eleccione al menos una respuesta. ,. Bablar como nosotros E. )ener la misma apariencia !ue nosotros C. Caminar como nosotros '. Bacer matem tica como nosotros #) Deg1n <7)rc7land, a)n()e el d)alismo no es la concepcin m.s dif)ndida dentro de la com)nidad filosfica 5 cientfica act)al, es sin em/argo la teora m.s pop)lari"ada so/re la mente- En opinin de este a)tor, esto se de/era a: ,. la influencia de la ciencia E. la influencia de las religiones universales C. la influencia de la filosofa &) Deg1n Dearle, el cond)ctismo metodolgico se de/ilit 5 f)e rec7a"ado por()e ,. los argumentos conductistas solo pudieron dar cuenta de las relaciones causales existente entre los estados mentales y los comportamientos E. confundi el 9tema9 de estudio de la psicologa 4la mente2 con 9las pruebas9 de la existencia de dicho tema 4el comportamiento humano2 C. solo el conductismo filosfico pudo proponer una explicacin plausible de cmo podan traducirse proposiciones sobre la mente en proposiciones sobre el comportamiento 0) Deg1n Dearle, los d)alistas act)ales se enfrentan a )n pro/lema adicional: la le5 fsica de la conser4acin de la materia, ()e dice ()e la cantidad de materiaWenerga en el )ni4erso es constante- Di el d)alismo es 4erdadero, ,. las explicaciones sobre el funcionamiento neurofisiolgico del cerebro y su estructura anatmica" seran falsas E. una ley fundamental de la fsica" como es la conservacin de la materia" sera falsa C. debera abandonarse la concepcin cl sica de la relacin materiaGenerga y adoptar una teora cu ntica probabilstica 2) <7)rc7land afirma ()e la diferencia entre el epifenomenismo 5 el d)alismo interaccionista de propiedades es ()e: ,. el epifenomenismo sostiene !ue las propiedades mentales no tienen efectos causales" mientras !ue el dualismo interaccionista de propiedades sostiene !ue si E. el epifenomenismo sostiene !ue las propiedades mentales tienen efectos casuales y el dualismo de propiedades dice !ue no C. el epifenomenismo sostiene !ue las propiedades mentales tienen solo efectos causales parciales" mientras !ue el dualismo interaccionista dice !ue no >) Deg1n Dearle, el cond)ctismo lgico se 7a enfrentado a dos tipos de o/6eciones: +eleccione al menos una respuesta. ,. las objeciones de car cter funcionalista E. las objeciones de sentido com*n C. las objeciones neurocientficas '. las objeciones de ndole tcnica

?) !ea la sig)iente afirmacin e indi()e a ()8 tipo de cond)ctismo pertenece: R!as explicaciones en psicologa de/en /asarse totalmente en nociones ()e sean directamente o/ser4a/les por todos o definidas operacionalmente en t8rminos de conceptos as o/ser4a/lesR,. pertenece al conductismo metodolgico E. pertenece al conductismo lgico C. pertenece al conductismo filosfico @) El principio de la cla)s)ra ca)sal de lo fsico, ()e forma parte del arg)mento de la cla)s)ra ca)sal, sostiene ()e: ,. en cual!uier momento !ue un estado fsico tenga una causa" tiene una causa fsica enteramente suficiente E. siempre !ue un estado fsico tenga una causa" tiene una causa no fsica enteramente suficiente 1A) El d)alismo de propiedades es, en sentido general, )na prop)esta: ,. indiferente al problema del reduccionismo E. reduccionista C. no reduccionista

!A FENPE RE%REDENPA<I'NA! $%ERNER S <A%- 1)


1) ;%or ()8 seg1n Xo7nson !aird las teoras psicolgicas del significado casi n)nca se oc)paron de la referencia= a2 &or!ue la referencia se explica mediante contenidos abstractos. b2 &or!ue esa pregunta re!uiere una respuesta biolgica: en la naturaleza no hay representacin sin evolucin. ) ;M)8 opina %erner respecto a la importancia de sa/er ()e 7ace a algo )na representacin= a2 Es simplemente una discusin filosfica con ninguna importancia para la psicologa. b2 7u hace a algo una representacin es imposible de definir as !ue no vale la pena discutir esta nocin. c2 &or!ue tiene implicaciones evolutivas palpables respecto de la manera !ue la mente del ni0o se despliega como sistemas de representaciones. #) En el experimento de Anderson9UoKer $1@>&) se le le5eron a s)s s)6etos 7istorias en la poda presentarse )n en)nciado como 8ste: 3!a c7ica f)e /esada por el m)c7ac7o-: Al final de la 7istoria se mostra/a /re4emente )no de los sig)ientes en)nciados: @. El muchacho bes a la chica. ?. La chica bes al muchacho. ;. El muchacho fue besado por la chica. F. La chica fue besada por el muchacho. Los sujetos deban apretar un botn 4)2 si uno de los enunciados 4@3F2 era verdadero y otro botn 4.2 si era falso. +e midi el tiempo de reaccin. Los sujetos respondan m s r pidamente al enunciado 4F2 !ue al 4@2. %ara %erner, este experimento pr)e/a ()e: a2 -o hay necesariamente representacin por!ue el enunciado !ue le leen es la forma m s com*n de expresar esta situacin y por eso los sujetos reaccionan m s r pido ante el mismo. b2 -o hay representacin en ning*n momento por!ue despus de los ? minutos esta preferencia desaparece. c2 Bay representacin por!ue la conducta posterior de elegir m s r pidamente el F2 solo puede explicarse por!ue el enunciado 9La chica fue besada por el muchacho9 fue representado de alg*n modo en los sujeto cuando le leyeron la historia. &) Deg1n %erner la diferencia entre representaciones primarias 5 sec)ndarias consiste en ()e: a2 las representaciones primarias no descansan en la relacin causal entre el mundo representado y el medio de representacin. b2 las representaciones primarias est n en estrecho contacto causal con el mundo c2 las representaciones primarias permiten plantear situaciones hipotticas. 0) B)me propona ()e el ente al c)al se refera )na representacin era: a2 un estado mental. b2 una relacin representacional c2 el referente real 2) %erner critica la nocin de representacin de B)me diciendo ()e a2 Bume confunde la nocin de relacin representacional con la de medio representacional b2 Bume confunde la nocin de representacin mental con la de representacin externa. c2 Bume confunde la nocin de contenido representacional con el medio representacional. >) <)ando se dice asimetra- ;M)8 se ()iere significar= a2 -o es lo mismo la representacin !ue los medios de representacin. b2 7ue no son iguales las representaciones entre s. c2 La imagen representa algo" pero eso 4representado2 no representa la imagen.

?) %ara %erner, c).les son las dos resp)estas /.sicas ()e responden a la preg)nta: ;M)8 es lo ()e 7ace ()e )na cosa se represente otra= a2 La naturalista y la estructuralista. b2 La intencionalista y estructuralista c2 Las respuestas intencionalista y la naturalista @) !a resp)esta nat)ralista a lo ()e constit)5e )na representacin mental dice: a2 El estatus representacional depende de !ue se las entienda o interprete como tales. b2 Bay procesos naturales" con independencia de la mente humana" !ue pueden otorgar estatus representacional. 1A) %ara %erner el significado de metarrepresentacin ()e el adopta es: a2 Lepresentacin de un contenido representacional !ue es una representacin 4ej. una representacin de un unicornio2 b2 Lepresentacin de una representacin en tanto representacin. c2 -o recursivo.

!A FENPE RE%REDENPA<I'NA! $%ERNER S <A%-

1) De ac)erdo con %erner, la opinin piagetiana de ()e los nios 7asta el ao de 4ida est.n 3ligados al estm)loR significa, entre otras cosas, ()e a2 &rocesan los estmulos integrando modalidades b2 &rocesan los estmulos no especificando la modalidad. c2 &rocesan los estmulos especficos a cada modalidad ) %erner s)giere ()e 7asta el ao los nios )tili"an )n 1nico modelo el c)al se diferencia, entre otras cosas, de los es()emas sensoriomotores piagetianos en ()e a2 )iene una modalidad especfica b2 -o tiene una modalidad especfica c2 )iene una modalidad sensorial perceptiva #) %erner s)giere ()e la imitacin facial en los nios pe()eos es )n e6emplo ()e p)ede ser explicado me6or con la post)lacin de )n modelo 1nico ()e con los es()emas sensoriomotores de %iaget por()e a2 Los modelos pueden integrar modalidades y esta tarea as lo re!uiere b2 Los modelos no pueden integrar modalidades pero si pueden dar cuenta de esta tarea c2 Los modelos pueden dar cuenta de la modalidad sensorial &) De ac)erdo con %erner, %iaget necesita considerar el 6)ego de sim)lacin, entre otras ra"ones por()e a2 anuncia el funcionamiento simblico del ni0o b2 permite comprender la funcin representativa c2 ayuda a entender los es!uemas sensorio3motores 0) %erner )sa la pala/ra 3metarrepresentacin: en el sentido a2 como representaciones secundarias b2 como representaciones de objetos no existentes c2 recursivo d2 como comentarios metarrepresentacionales 2) Deg1n %erner, en el 6)ego de sim)lacin el nio de/e tener alg1n grado de conciencia de a2 la funcin desempe0ada en la actividad l*dica b2 la discrepancia con la realidad c2 la funcin desempe0ada por el mismo ni0o >) %ara %erner, la imitacin facial ()e 7ace el reci8n nacido es a2 metarrepresentacin b2 sensorio3motora c2 un modelo *nico !ue se actualiza d2 dos modelos !ue no se actualizan ?) El 6)ego de sim)lacin es para %erner: a2 simblica b2 actuar como si c2 metarrepresentacional

!A FENPE RE%REDENPA<I'NA! $%ERNER)


1) Eno de los criterios ()e %erner anali"a para comprender lo mental es el de 3experiencia interior:- De 8l dir. ()e: a2 es un criterio *til" pero no alcanza para comprender la mente. En particular no proporciona una buena solucin al problema de las Jotras mentesK. b2 es la fuente primaria de nuestra comprensin de la menteM esta nos permite atribuir estados mentales a otros en base a nuestra propia experiencia c2 la conciencia de la experiencia interior no tiene ning*n rol en la comprensin de la mente. ) Deg1n el criterio de diferencia entre lo fsico 5 lo mental ()e %erner denomina 3constr)ctos terico:, los estados mentales c)mplen )na f)ncin explicati4a en n)estra psicologa de sentido com1n- Esto significa ()e: a2 los estados mentales forman parte de constructos tericos" de una teora de la mente !ue explica cmo nos comportamos" y !ue define el significado independiente de cada uno de los trminos mentales. b2 las creencias y deseos no ad!uieren su significado por definicin sino por el papel !ue cumplen en la explicacin de la conducta" de tal modo !ue atribuir estados mentales es atribuir una Jteora de la menteK. c2 las creencias y deseos ad!uieren su significado por definicin" explicitado a!uello a lo !ue refieren. #) El tercer criterio ()e %erner es la 3intencionalidad: o 3inexistencia intencional:- Esta 1ltima expresin refiere a: a2 al modo en !ue los objetos mentales 4contenido mental2 existe. b2 al hecho de !ue pueda referir a algo !ue no existe" en cuyo caso vuelve falso el contenido del estado mental en cuestin. c2 !ue a!uello a lo !ue se dirigen los estados de actitud proposicional no existe. &) Deg1n %erner podemos reconocer tres criterios por los c)ales la inexistencia intencional esta/lece la diferencia entre lo fsico 5 lo mental: a2 no existencia" metarrepresentacin y aspectualidad. b2 representacin errnea" intencionalidad" aspectualidad y no existencia. c2 no existencia" aspectualidad y representacin errnea. 0) Deg1n %erner 3A1n c)ando )n acto mental tienda a algo ()e realmente existe, como en el caso del conocimiento, la intencionalidad mental difiere de la relacin fsica:- <on esto est. ()eriendo sealar ()e: a2 aun cuando el estado mental se dirige a o apunta a un objeto existente" lo hace solo en relacin a algunos aspectos del objeto b2 el estado mental al igual !ue la relacin fsica implica el objeto en todos sus aspectos c2 el estado mental implica al objeto al !ue refiere en todos sus aspectos mientras !ue la relacin fsica no

!A FENPE RE%REDENPA<I'NA! $%ERNER)


1) !a consec)encia general ()e p)ede extraerse del an.lisis ()e 7ace %erner so/re la psicologa de sentido com1n es ()e: a2 Bay mucho !ue un ni0o puede comprender sobre la mente sin comprender la representacin b2 El ni0o necesita comprender la representacin para convertirse en psiclogo de sentido com*n c2 El ni0o no necesita comprender la representacin para convertirse en psiclogo de sentido com*n d2 -inguna de las anteriores ) Indi()e ()8 contestara %erner frente al sig)iente c)estionamiento: 3de ac)erdo con el sentido com1n, ;la mente es representacional=: a2 -o" no es representacional b2 +" la mente es enteramente representacional c2 ,lgunos de sus aspectos lo son mientras otros" no d2 +" es un sistema representacional de procesamiento de informacin #) %erner )sa la distincin entre Y4er epist8micoZ 5 Y4er no epist8micoZ para mostrar ()e: a2 El sentido com*n no guarda relacin alguna con el lenguaje natural de las personas b2 El sentido com*n asume las distinciones !ue permite nuestro lenguaje natural c2 -inguna de las anteriores d2 El sentido com*n puede ignorar algunos aspectos acerca de la mente !ue est n contemplados en algunas distinciones del lenguaje natural

INPEN<I'NA!IDAD $DEAR!E)
1) ;M)8 es la intencionalidad=: a2 La propiedad !ue tienen algunos estados mentales de referirse a objetos y situaciones en el mundo b2 La capacidad de algunos estados mentales !ue nos permite dirigirnos hacia objetos y situaciones del mundo para satisfacer nuestras necesidades y deseos c2 El rasgo fundamental !ue distingue una accin de un mero movimiento corporal ) ;M)8 entiende Dearle por el 3modo psicolgico: de )n estado intencional=: a2 El tipo de actitud !ue tenemos frente al contenido proposicional del estado y !ue de este modo lo define b2 La manera en !ue el estado se presenta internamente a a!uel !ue tiene el estado c2 ,!uello !ue determina la referencia del estado #) !o ()e Dearle llama 3trasfondo: es: a2 La red de estas mentales !ue define el contenido de un estado mental en particular b2 La base neurobiolgica en *ltima instancia responsable de la intencionalidad de los estados mentales c2 (n conjunto de aptitudes" disposiciones y capacidades b sicas para la accin en el mundo &) Ena frase como 3la l)na 4ia6a alrededor de la tierra: pasa la pr)e/a de extensionalidad si: a2 es imposible sustituir el contenido del enunciado por otro contenido de igual referente sin prdida de significado b2 en ocasiones se puede sustituir el contenido del enunciado por otro contenido de igual referente por ejemplo Jel satlite de la tierra viaja alrededor de esta 4la tierra2K c2 es posible sustituir el contenido del enunciado por otro contenido de igual referente" por ejemplo por Jel satlite de la tierra viaja alrededor de esta 4la tierra2K" sin prdida de significado 0) A diferencia de )n en)nciado ()e pasa la pr)e/a de extensionalidad, )n en)nciado intensional a2 puede remitir a algo !ue no existe b2 refiere *nicamente a algo !ue no existe c2 no podra hacer alusin a algo !ue no existe 2) Respecto de la diferencia entre intensionalidad 5 la intencionalidad, Dearle sostiene ()e: a2 el hecho de !ue los enunciados intencionales sean a menudo intensionales es relevante para comprender la naturaleza de la intencionalidad de los estados mentales b2 lo !ue es intensional es la representacin del estado intencional no la intencin misma c2 lo !ue explica la particularidad de los fenmenos intencionales se explica en parte por!ue se expresan mediante enunciados intencionales >) !a intencionalidad es: a2 una propiedad !ue solo tienen los enunciados b2 una propiedad !ue tienen algunos estados mentales y ciertos enunciados c2 una propiedad !ue solo tienen los estados mentales

%R'U!EFA DEF[NPI<'
1) Deg1n <7)rc7land, para el <ond)ctismo los elementos mentales como las creencias 5 los deseos son: ,. conductas explicitas E. comportamientos acompa0ados de afectos C. disposiciones fsicas para el movimiento corporal '. estados subjetivos !ue acompa0an a los comportamientos ) %artiendo de la exposicin de !oKe, se afirma ()e la concepcin realista de los estados mentales se disting)e de la concepcin instr)mentalista, en ()e la primera: ,. acepta el uso del trmino estados mentales solo a nivel terico E. acepta la existencia de los estados mentales adoptando una postura materialista reduccionista C. acepta la existencia de la intencionalidad como rasgo de la conciencia '. acepta la existencia de los estados mentales sin discutir si estos son idnticos a los estados fsicos #) <7)rc7land afirma ()e no es posi/le definir 5 clasificar los estados mentales por RostensinR 5a ()e los estados mentales o psicolgicos: ,. tienen cualidades fenomnicas pero no tienen eficacia causal E. solo tienen caracteres cualitativos y subjetivos !ue no permiten construir teoras cientficas C. no tienen car cter cualitativo ni !ualia !ue los distingan entre s. &) Deg1n !oKe, los materialistas eliminati4os rec7a"an el 4oca/)lario de las actit)des proposicionales por()e consideran ()e pertenece a )na teora pre cientfica de la menteDerdadero .also 0) !oKe afirma ()e existe )na conciencia fenom8nica 5 )na conciencia percepti4a- Define la <onciencia %ercepti4a como el rasgo distinti4o de los estados c)alitati4os de experienciaDerdadero .also 2) !a intencionalidad es considerada como el rasgo ()e diferencia los estados mentales de otros fenmenos, 5a ()e se refiere a: ,. el sentido o significado !ue posee un trmino E. la capacidad de los estados mentales de ser sobre otra cosa C. los motivos !ue nos llevan a realizar ciertas acciones >) En relacin a la Intencionalidad, Dearle seala ()e existen 4arios pro/lemas, ()e re()ieren resp)estas- Deale todos los pro/lemas a los ()e 7ace referencia Dearle: #$eleccione al menos una respuesta% ,. Cmo se conectan mis creencias y pensamientos con los objetos en ellos representados E. Cmo es posible !ue en ciertas ocasiones pueda pensar en objetos !ue no existen 4ej: los duendes2 C. Cmo es posible !ue utilice palabras !ue no tienen referencias exactas en la realidad '. Cmo es posible !ue la intencionalidad de la mente se expli!ue por la intencionalidad del lenguaje E. Cmo puedo estar seguro de !ue mis pensamientos apuntan a cierto objeto y no a otro 4ej: a Csar y no a ,ugusto2 ?) c)ando <7)rc7land 7a/la de la teora retic)lar del significado, se refiere a: ,. aprendemos los significados de las palabras de nuestro idioma" y este sirve como una red sem ntica sobre la cual construimos las teoras cientficas !ue predicen y explican el comportamiento humano

E. los trminos utilizados para referirse a los estados mentales obtienen su significado de un marco de referencia incorporado a nuestros conocimientos corrientes 4psicologa popular2 C. los seres humanos en general utilizamos un lenguaje privado con trminos !ue se refieren al vocabulario de las sensaciones

%R'U!EFA DEF[NPI<' $<BER<B!AND)


1) !o ()e llamamos psicologa corriente o pop)lar es: a2 un marco de referencia !ue adapta los trminos mentales cientficos a modos cotidianos de explicacin. b2 un marco de referencia terico" !ue incluye leyes o generalizaciones" explicaciones y predicciones acerca de cmo los seres humanos actuamos y funcionamos. c2 un marco de referencia !ue los hablantes cotidianos usamos en situaciones especiales para explicar conductas fuera de lo normal ) Deg1n <7)rc7land el de/ate acerca de la relacin entre psicologa cientfica 5 psicologa pop)lar: a2 implica una tensin entre dos modos de abordar el mbito de lo mental con criterios epistemolgicos diferentes" por tanto" irreductibles uno a otro. b2 no depende de las investigaciones empricas pues es un debate conceptual c2 es una cuestin emprica !ue resolver el avance en las investigaciones en neurociencias y psicologa cognitiva. #) Alg)nos estados mentales se denominan estados de actit)d proposicional: a2 por!ue expresan una actitud hacia una proposicin cuyo contenido es el mismo del estado en cuestin. b2 por!ue expresan un contenido proposicional. c2 por!ue expresan un estado mental. &) En ocasiones se 7a pensado ()e la intencionalidad de los estados de actit)d proposicional $esto es, el 7ec7o de ()e 3sean acerca de algo:): a2 es un rasgo !ue poseen todos los estados mentales concientes como creer" desear" sentir" esperar" percibir" etc. b2 es un rasgo !ue podemos atribuir tanto a entidades fsicas como mentales c2 es un rasgo !ue distingue lo mental de lo meramente fsico. 0) Ena definicin ostensi4a consiste en: a2 'ar una descripcin completa de los rasgos de a!uello !ue se define b2 Caracterizar el objeto definido en base a los rasgos !ue sean visibles por !uienes realizan la definicin c2 /ostrar directamente un caso del tipo de cosas !ue se define 2) El arg)mento del leng)a6e pri4ado: a2 /uestra la posibilidad en principio de un lenguaje privado b2 /uestra la imposibilidad de un lenguaje absolutamente privado c2 /uestra !ue el de los trminos psicolgicos es un caso de lenguaje privado >) !a teora retic)lar del significado dice ()e: a2 El significado deriva de la red de relaciones !ue tiene un trmino respecto de otros trminos *nicamente referidos a circunstancias y conductas observables b2 El significado se da en una red intricada de capacidades pr cticas para el desempe0o exitoso en el mundo c2 El significado deriva del lugar !ue un trmino ocupa en un sistema de leyes ?) De ac)erdo al modelo nomolgico9ded)cti4o, explicar es: a2 'ar un argumento a la mejor explicacin b2 &redecir !ue una ley natural se aplica a un cierto fenmeno c2 'ar un argumento !ue contiene en sus premisas una ley y condiciones iniciales

INPEN<I'NA!IDAD 9 <BER<B!AND
1 De ac)erdo con <7)rc7land, las definiciones ostensi4as com)nican el significado de )n t8rmino mostrando simplemente )n caso concreto de a()ello ()e se intenta definir- %ara cierta posicin, existen ciertos t8rminos psicolgicos c)5o significado solo p)ede conocerse por medio de la ostensin, a partir de lo ()e se experimenta 5 se perci/e de ellosA 8sta post)ra se la denomina: a. >stensin 'irecta b. Conductismo .ilosfico. c. Criterio -ormal d. +olipsismo +em ntico. De todas las estrategias filosficas para explicar la INPEN<I'NA!IDAD de los fenmenos mentales, Dearle se apo5a en la Red)ccin UiolgicaEsta sostiene ()e: a. Los sistemas no3biolgicos" como las m !uinas" podran tener estados intencionales y ser explicados en trminos biolgicos. b. (n estado transporta informacin sobre otro en el grado en !ue depende de ese por una relacin causal3determinista" y en ste sentido el rasgo de ser Jsobre algoK se establece en todas las relaciones causales. c. 7ue la $ntencionalidad es caracterstica de los sistemas biolgicos y por lo tanto para explicarla necesitamos de una teora biolgica. d. Cuando caracterizamos a un sistema" natural o artificial" en trminos de creencias y deseos" adoptamos una postura intencional !ue permite explicar y predecir la conducta de los dem s. # Deg1n <7)rc7land )no de los %RIN<I%A!ED pro/lemas de la Pesis <ond)ctista so/re como ad()iere significado )n t8rmino es: a. 7ue atribuye un papel insignificante a los !ualia de los estados mentales b. 7ue se re!uiere de una lista infinitamente larga de condicionales !ue ayudan a definir un estado mental especfico. c. 7ue existen conductas no observables" como los estados mentales" !ue no pueden ser definidos por stos. d. 7ue podra ser sustituido por la )esis )erica Leticular" mantenindose igualmente su concepcin. & !a Intencionalidad de los t8rminos se refiere a: a. +e utiliza como otra forma de denominar el .allo de +ustitutividad. b. -inguna es correcta. c. El sentido al cual se alude con el trmino utilizado" es decir su significado. d. La capacidad de tener un contenido y tratar acerca de algo distinto de s mismo. 0 !os defensores de la Pesis Perico Retic)lar s)elen sostener ()e los ()alia tienen )na significacin epistemolgica pero est.n despro4istos de significacin sem.ntica para los t8rminos de )n leng)a6e interno- Es decir, ()e a pesar de aportar conocimiento so/re el estado al c)al se al)de, los ()alia no son f)ndamentales para conocer el significado de )n t8rmino, 5a ()e no aportan )n contenido sem.ntico al mismoDerdadero .also 2 !a post)ra Instr)mentalista es a()ella ()e sostiene ()e las entidades post)ladas para explicar )n fenmeno p)eden )tili"arse si 5 solo s son 4erdaderas 5 s) existencia es s)scepti/le de compro/acin cientficaDerdadero .also

> Ena de las estrategias filosficas ()e intentan explicar la Intencionalidad, presenta las Actit)des %roposicionales como la /ase para ela/orar )na explicacin cientfica de cmo opera la mente, sosteniendo ()e la mente tra/a6a con sm/olos ()e se manip)lan de ac)erdo con ciertas reglas- Iarias son las o/6eciones a 8sta concepcin- Farca opciones ()e consideres correctas: a. La mente no podra almacenar el conjunto de oraciones !ue se re!uieren para cada accin !ue realizamos y adem s" stas dependen de un contexto !ue va m s all de las posibilidades de programacin. b. 7ue la conducta racional del hombre no puede explicarse en trminos de manipulacin de smbolos. c. El intento de explicar la intencionalidad de los estados mentales apelando a la existencia de relaciones causales ente estos estados y objetos externos" ya !ue esto impide explicar a!uellos !ue se refieren a objetos no existentes.

<'NPENID' FENPA! $!'NE)


1 !a explicacin externista del contenido mental s)pone ()e el contenido de las actit)des proposicionales se determina por las relaciones del s)6eto con s) entorno fsico- De contrapone a la nocin de: a. representaciones: el contenido representa al mundo como algo !ue es de una cierta manera b. contenido reducido: el contenido es independiente de las relaciones contextuales c. indexicalidad: el lenguaje contiene referencias a tiempo" lugar y objetos concretos !oKe afirma ()e no es f.cil de explicar el modo en ()e el contenido mental podra ser ca)salmente pertinente, dado ()e: a. las proposiciones utilizadas pertenecen a la psicologa popular b. las proposiciones !ue constituyen dicho contenido son entidades abstractas c. las actitudes de una persona se determinan por su relacin con el entorno # !oKe se m)estra a fa4or de )na Rconcepcin ampliaR del contenido mental de/ido a ()e: a. los contenidos de las proposiciones !ue las personas utilizan determinan sus actitudes hacia el entorno b. los contenidos de los estados actitudinales se determinan por las relaciones entre la persona y su entorno c. las proposiciones !ue constituyen dicho contenido son entidades abstractas & Deg1n !oKe, n)estro pensamiento es implcitamente ind8xico- !a indexicalidad implica ()e: a. el contenido proposicional es causalmente dependiente b. el contenido proposicional es dependiente de las representaciones causales c. el contenido proposicional es dependiente del contexto del sujeto 0 !oKe considera ()e los enfo()es nat)ralistas tienen dific)ltades para explicar ()8 es lo ()e otorga )n contenido especfico a )n estado de actit)d, 5a ()e: a. no son aplicables a las representaciones mentales presentes en todos los tipos de estados mentales" sino solo a los de actitud b. no son f cilmente aplicables a las representaciones mentales presentes en los estados de actitud de sujetos inteligentes c. son solo aplicables a las representaciones mentales presentes en los estados de actitud de sujetos inteligentes 2 !a concepcin realista de los estados mentales se disting)e de la concepcin instr)mentalista en ()e la primera: a. acepta la existencia de la intencionalidad como rasgo de la conciencia b. acepta la existencia de los estados mentales sin discutir si estos son idnticos a los estados fsicos c. acepta la existencia de los estados mentales adoptando una postura materialista reduccionista d. acepta el uso del trmino 9estados mentales9 solo a nivel terico > !oKe afirma ()e los enfo()es nat)ralistas tienen dific)ltades para explicar ()8 es lo ()e otorga )n contenido especfico a )n estado de actit)d- Dic7os enfo()es son: a. la teora causal y la teora adverbial b. la teora causal y la teora sensorialista c. la teora causal y la teora teleolgica ? Respecto al origen del contenido mental, !oKe anali"a dos enfo()es: el ca)sal 5 el teleolgico- !a explicacin teleolgica s)pone: a. !ue solo los seres humanos pueden poseer estados mentales por!ue solo estos tienen objetivos y finalidades

b. !ue los estados mentales son rasgos evolutivamente ad!uiridos y funciones especificas de organismos biolgicos c. !ue los estados mentales son representaciones del mundo como algo !ue contiene un estado de cosas 4E2" tal !ue E es la causa de esos estados @ Deg1n !oKe, al anali"ar el contenido de las actit)des proposicionales, es preciso anali"ar: a. su pertinencia causal" especificidad y origen b. su eficacia causal" externismo y contenido reducido c. su pertinencia causal" sus representaciones y teleologa 1A Deg1n !oKe, las proposiciones no poseen eficacia ca)sal, pero si Rpertinencia ca)salR en c)anto al contenido proposicional de )n estado de actit)d proposicional- Don pertinentes ca)salmente por()e a. su referencia tiene un papel no redundante en la explicacin causal b. tiene poder de ser causa y efecto del fenmeno aludido c. son an logas a los n*meros en la resolucin de problemas

<'NPENID' FENPA! $!'NE) 9


1) !oKe se m)estra a fa4or de )na concepcin amplia del contenido mental de/ido a ()e los contenidos de las proposiciones ()e las personas )tili"an determinan s)s actit)des 7acia el entornoDerdadero .also ) Deg1n !oKe, n)estro pensamiento es implcitamente ind8xico- !a indexicalidad implica ()e el contenido proposicional es dependiente del contexto del s)6etoDerdadero .also

#) Al afirmar ()e todo estado mental es dependiente de las relaciones del s)6eto con s) entorno, !oKe o/6eta: a. al materialismo eliminativo por su rechazo de la existencia de los estados mentales b. al emergentismo por su enfo!ue teleolgico de la representacin mental c. al dualismo cartesiano en su concepcin individualista o internista de la mente &) !os defensores de la teora ca)sal intentan sol)cionar el pro/lema de la representacin errnea introd)ciendo la nocin de: a. contenido estrecho b. normalidad c. teleologa 0) %ara !oKe )na proposicin es: a. una entidad lingCstica concreta: una cadena de palabras con una organizacin gramatical correcta b. el significado de un enunciado o el contenido de una actitud proposicional c. la aseveracin de una oracin realizada por los usuarios de una lengua 2) !oKe afirma ()e no es f.cil de explicar el modo en ()e el contenido mental podra ser ca)salmente pertinente, dado ()e: a. las proposiciones utilizadas pertenecen a la psicologa popular b. las proposiciones !ue constituyen dicho contenido son entidades abstractas c. las actitudes de una persona se determinan por su relacin con el entorno >) !a teora ca)sal estrec7a introd)ce )na nocin de normalidad teleolgica para sol)cionar el pro/lema de la representacin errneaDerdadero .also

?) <)ando !oKe afirma ()e Yn)estras mentes no son recipientes de pensamientos a)tocontenidos 5 7erm8ticamente cerrados, sino ()e se extienden 7acia el entorno fsicoZ, est. reconociendo ()e n)estros estados mentales dependen 9 en s)s contenidos 5 en s) identidad 9 de c)ales sean las cosas ()e contiene el entornoDerdadero .also

@) Respecto al origen del contenido mental, !oKe anali"a dos enfo()esEno de ellos es la explicacin ca)salH esta s)pone ()e solo los seres 7)manos p)eden poseer estados mentales por()e solo estos tienen o/6eti4os 5 finalidadesDerdadero .also

1A) !a concepcin realista de los estados mentales se disting)e de la concepcin instr)mentalista en ()e la primera: a. acepta la existencia de los estados mentales sin discutir si estos son idnticos a los estados fsicos b. acepta la existencia de los estados mentales adoptando una postura materialista reduccionista c. acepta la existencia de la intencionalidad como rasgo de la conciencia

DENDA<IVN \ A%ARIEN<IA $!'NE)


1) A 4eces )na persona a la ()e se le 7a amp)tado )na pierna contin)a sintiendo dolores 5 otras sensaciones LenL la pierna amp)tada- Este fenmeno del miem/ro fantasma se )tili"a como )n arg)mento a fa4or de: a. la teora de los datos sensoriales b. la teora causal de las percepciones c. el realismo directo d. la teora adverbial de las sensaciones ) Deg1n !oKe, las experiencias percept)ales no p)eden carecer de contenido proposicionalDerdadero .also

#) !oKe rec7a"a como teoras explicati4as de las percepciones tanto al enfo()e comp)tacional como al enfo()e ecolgico, principalmente por considerar ()e: a. ninguno de ambos considera la funcin adaptativa !ue cumplen los sistemas perceptuales b. ambos definen los sistemas perceptuales como sistemas de procesamiento de informacin interna c. ninguno de ambos puede explicar los rasgos cualitativos de las experiencias perceptuales &) !oKe afirma, en relacin a la percepcin, ()e el enfo()e comp)tacional es implcitamente internista, mientras ()e el ecolgico es implcitamente externista- Esto significa ()e el enfo()e comp)tacional 7ace 7incapi8 en la relacin de dependencia entre los estados mentales del s)6eto 5 s) contextoDerdadero .also

0) Desde el enfo()e comp)tacional, se definen los estados percept)ales como sistemas de procesamiento de informacin an.logos a comp)tadoras- Deg1n el modelo de Farr, la percepcin de los o/6etos es indirecta por()e: a. el sistema no construye representaciones de los objetos del entorno b. el sistema construye representaciones de los objetos del entorno c. el sistema facilita la conducta adaptativa 2) !a expresin 3Ieo ()e el .r/ol esta ante la casa:, es: a. un juicio perceptual b. solo un estado de sensacin c. solo un estado de actitud proposicional >) !os estados de sensacin, seg1n !oKe, tienen contenido proposicional como los estados percept)alesDerdadero .also ?) %or ()8 7a/ramos de decir ()e los s)6etos con 34isin ciega: realmente 4en cosas ()e se enc)entran en las regiones ciegas de s)s campos 4is)ales=: a. &or!ue es imposible determinar !ue regin del cortex visual est da0ada b. &or!ue habitualmente no se considera esta informacin como relevante c. &or!ue las conjeturas !ue realizan acerca de la presencia o ausencia de estmulos en las regiones ciegas de su campo visual suelen ser correctas @) Deg1n el enfo()e comp)tacional, en il)siones 4is)ales como las prop)estas por F)ller9!5er 34emos: fig)ras distintas por()e no podemos corregir la informacin ()e nos en4a n)estra sistema 4is)alH esto se de/e

a ()e los procesos de alto ni4el no p)eden modificar los procesamientos efect)ados en ni4eles s)/informacionales o sistemas mod)laresDerdadero .also

1A) El arg)mento a partir de la Il)sin es )n arg)mento a fa4or de: a. la teora causal $ncorrecto b. la teora adverbial de las sensaciones c. la teora de los datos sensoriales d. el realismo directo

DENDA<IVN \ A%ARIEN<IA $!'NE) 9


1) De ac)erdo con !oKe 7a/ra )n )so epist8mico de RparecerR, ()e consiste en a2 un hablante !ue describe caractersticas de la forma en la cual percibe un objeto b2 un hablante !ue expresa alguna propiedad de los datos sensoriales c2 un hablante !ue matiza o aten*a una afirmacin de conocimiento !ue l hace ) !as sensaciones tienen: a2 +lo contenido conceptual b2 slo contenido cualitativo c2 contenido cualitativo y contenido conceptual #) De ac)erdo con !oKe 7a/ra )n )so Rfenom8nicoR de parecer c)ando a2 se est tratando de transmitir algo acerca del objeto !ue se est percibiendo b2 se est tratando de transmitir algo acerca de las cualidades !ue conforman a!uello !ue se est percibiendo c2 se est tratando de transmitir algo acerca del car cter cualitativo o fenomnico de las propias experiencias precept*ales &) De ac)erdo con la teora ad4er/ial, se de/era poder parafrasear )na expresin como LDiento )n dolor f)erte en la espaldaL por otra en la c)al a2 el sustantivo original sea reemplazado por un adverbio y el adjetivo por un sinnimo b2 el sustantivo original sea reemplazo por otro sustantivo" pero sin las implicaciones metafsicas del primero" y el adjetivo por un adverbio c2 el sustantivo original sea reemplazado por un verbo y el adjetivo por un adverbio 0) A 4eces )na persona a la ()e se le 7a amp)tado )na pierna contin)a sintiendo dolores 5 otras sensaciones LenL la pierna amp)tada- Este fenmeno del miem/ro fantasma se )tili"a como )n arg)mento a fa4or de: a. la teora de los datos sensoriales b. la teora causal de las percepciones c. el realismo directo d. la teora adverbial de las sensaciones 2) Deg1n !oKe, las experiencias percept)ales no p)eden carecer de contenido proposicionalDerdadero .also

>) !oKe rec7a"a como teoras explicati4as de las percepciones tanto al enfo()e comp)tacional como al enfo()e ecolgico, principalmente por considerar ()e: a. ninguno de ambos considera la funcin adaptativa !ue cumplen los sistemas perceptuales

b. ambos definen los sistemas perceptuales como sistemas de procesamiento de informacin interna c. ninguno de ambos puede explicar los rasgos cualitativos de las experiencias perceptuales ?) !oKe afirma, en relacin a la percepcin, ()e el enfo()e comp)tacional es implcitamente internista, mientras ()e el ecolgico es implcitamente externista- Esto significa ()e el enfo()e comp)tacional 7ace 7incapi8 en la relacin de dependencia entre los estados mentales del s)6eto 5 s) contextoDerdadero .also

@) Desde el enfo()e comp)tacional, se definen los estados percept)ales como sistemas de procesamiento de informacin an.logos a comp)tadoras- Deg1n el modelo de Farr, la percepcin de los o/6etos es indirecta por()e: a. el sistema no construye representaciones de los objetos del entorno b. el sistema construye representaciones de los objetos del entorno c. el sistema facilita la conducta adaptativa 1A) !a expresin 3Ieo ()e el .r/ol esta ante la casa:, es: a. un juicio perceptual b. solo un estado de sensacin c. solo un estado de actitud proposicional 11) !os estados de sensacin, seg1n !oKe, tienen contenido proposicional como los estados percept)alesDerdadero .also

1 ) %or ()8 7a/ramos de decir ()e los s)6etos con 34isin ciega: realmente 4en cosas ()e se enc)entran en las regiones ciegas de s)s campos 4is)ales=: a. &or!ue es imposible determinar !u regin del cortex visual est da0ada b. &or!ue habitualmente no se considera esta informacin como relevante c. &or!ue las conjeturas !ue realizan acerca de la presencia o ausencia de estmulos en las regiones ciegas de su campo visual suelen ser correctas 1#) Deg1n el enfo()e comp)tacional, en il)siones 4is)ales como las prop)estas por F)ller9!5er 34emos: fig)ras distintas por()e no podemos corregir la informacin ()e nos en4a n)estra sistema 4is)alH esto se de/e a ()e los procesos de alto ni4el no p)eden modificar los procesamientos efect)ados en ni4eles s)/informacionales o sistemas mod)laresDerdadero .also

1&) El arg)mento a partir de la Il)sin es )n arg)mento a fa4or de: a. la teora causal b. la teora adverbial de las sensaciones c. la teora de los datos sensoriales d. el realismo directo

DENDA<IVN \ %ER<E%<IVN $!'NE)


1 Damos )n R)so fenom8nicoR a los 4er/os RparecerR 5 RsentirR c)ando: a. )ratamos de transmitir algo acerca del car cter cualitativo de las propias experiencias perceptuales" sin comprometernos con afirmar !ue se trate de un rasgo del objeto externo percibido b. )ratamos de transmitir algo acerca del car cter cualitativo de las propias experiencias perceptuales" !ue creemos se corresponde con un objeto externo percibido c. $ntentamos matizar o suavizar una afirmacin acerca de un rasgo de un objeto externo !ue estamos percibiendo El en)nciado RX)an 4e )na man"ana 4erde so/re )na mesa si 5 solo si so/re la mesa realmente 7a5 )na man"ana 4erdeR e6emplifica las percepciones seg1n: a. el enfo!ue computacional b. la teora disyuntiva c. las teoras causalistas d. el enfo!ue ecolgico # !a teora ad4er/ial de las sensaciones se caracteri"a por: a. Considerar de manera literal la analoga entre los reportes de sensaciones como J)engo una aguda sensacin de hambre en mi estmagoK y J)engo una silla nueva en el comedorK b. $nterpretar los reportes de sensaciones de manera diferente al terico de los datos sensoriales" para evitar la tentacin de pensar en las sensaciones como objetos mentales con propiedades an logas a las de los objetos externos y espacialmente situados. c. Erindar una interpretacin de nuestros reportes de sensaciones !ue nos permita por completo el nfasis en la experiencia subjetiva puesto por la )eora de los 'atos +ensoriales & !os estados percept)ales, como los estados sensoriales, tienen rasgos c)alitati4os, 5 al ig)al ()e las actit)des proposicionales, tienen: a. juicios perceptuales b. estados actitudinales c. contenido conceptual 0 !oKe afirma ()e sensaciones 5 percepciones tienen al menos )n rasgo similar, ()e de/e tenerse en c)enta a la 7ora de explicarlos cientficamente- Am/os tipos de estados mentales: a. implican caractersticas cualitativas b. hacen inferencias sobre 9apariencias9 c. tienen siempre el mismo contenido conceptual 2 %erci/imos o/6etos en 4irt)d de los Lefectos sensorialesL ()e nos prod)cen- Dic7os efectos dependen de lo ()e se denomina Lc)alidades primariasL 5 Lc)alidades sec)ndariasL, o sensi/les com)nes 5 propios- !a diferencia, seg1n los tericos de los datos sensoriales, radica en ()e: a. las cualidades primarias son innatas" y las secundarias" aprendidas b. las cualidades primarias est n en los objetos" y las secundarias" en nosotros c. las cualidades primarias implican conceptos primitivos" y las secundarias" conceptos construidos > Deg1n la teora de los datos sensoriales:
a. &ercibimos de manera directa los datos sensoriales" !ue son cualidades secundarias de los objetos

b. )enemos percepciones directas objetos internos !ue poseen ciertas propiedades sensoriales !ue el objeto externo parece poseer )enemos percepciones directas objetos internos !ue poseen ciertas propiedades sensoriales !ue el objeto externo parece poseer

c. -uestras percepciones de los objetos externos son indirectas y aparecen mediadas por ciertos rasgos cualitativos de nuestra experiencia ? Deg1n la teora dis5)nti4a, no 7a5 elementos com)nes entre percepcin 4erdica 5 al)cinacin- Esto es por()e percepcin 4erdica nos pone en contacto directo con )n o/6eto externo, pero en la al)cinacin---: a. hay un objeto externo !ue distorsionamos por las condiciones contextuales b. hay un objeto externo !ue distorsionamos por nuestras creencias c. no hay un objeto externo" si no un 9me parece !ue9 4apariencia2 @ Deg1n !oKe, las experiencias percept)ales se disting)en de los 6)icios percept)ales por()e las primeras son m.s ricas ()e los seg)ndos en relacin al entornoH pero tam/i8n por()e: a. tener una experiencia perceptual no e!uivale a !uerer expresar o a poder expresar algo sobre la misma b. tener una experiencia perceptual implica un rasgo cualitativo irreductible y primitivo c. tener una experiencia perceptual refiere a contenido tanto conceptual como no3 conceptual 1A Deg1n !oKe, los rasgos c)alitati4os de las experiencias percept)ales se relacionan con los contenidos concept)ales de los estados percept)ales, por()e: a. los contenidos conceptuales son conceptos observacionales b. las apariencias transmiten aspectos cualitativos experienciales c. los rasgos cualitativos son cadenas causales 11 Desde el enfo()e comp)tacional, se definen los estados percept)ales como sistemas de procesamiento de informacin an.logos a comp)tadoras- Deg1n el modelo de Farr, la percepcin de los o/6etos es indirecta por()e: a. el sistema facilita la conducta adaptativa b. el sistema construye representaciones de los objetos c. el sistema describe la tarea a realizar en relacin al entorno 1 El arg)mento de la il)sin, de los tericos ()e post)lan datos sensoriales, falla al 7acer )na peticin de principio- As)me ()e: a. al observar un objeto" somos concientes de un objeto interno !ue no tiene relacin con ning*n objeto real b. al observar un objeto" somos concientes directamente de un objeto 4interno2 !ue tiene realmente la propiedad !ue el objeto parece tener c. al observar un objeto" somos concientes de la existencia de un objeto aun este ya no exista 1# !a diferencia entre el enfo()e comp)tacional 5 el enfo()e ecolgico de las percepciones est. en ()e para el primero la percepcin es indirecta mientras ()e para el seg)ndo, la percepcin es directa- M)e sea RdirectaR implica ()e: a. las estructuras fsicas del sistema est n directamente implicadas en el procesamiento de la informacin b. la informacin est disponible en el entorno y la usamos sin representarla c. percibimos por!ue eso favorece la supervivencia de los seres vivos 1& !os estados percept)ales, como los estados sensoriales, tienen rasgos c)alitati4os, 5 al ig)al ()e las actit)des proposicionales, tienen: a. contenido conceptual b. juicios perceptuales c. estados actitudinales 10 !os est)dios con s)6etos ()e tienen R4isin ciegaR m)estran ()e tener sanos los o6os no es s)ficiente para ()e se p)eda 4er- Deg1n !oKe, estos s)6etos afirman Rno 4erR por()e:

a. falla el procesamiento de la informacin al nivel de los juicios perceptuales b. falla el procesamiento de la informacin visual al nivel de la conciencia c. falla el procesamiento de la informacin visual a nivel de las actitudes proposicionales 12 No tenemos sensaciones 5 percepciones en relacin a partc)las s)/atmicas como los electrones por()e: a. no podemos construir conceptos observacionales sobre ellos b. no podemos hacer juicios valorativos sobre su apariencia c. no podemos relacionarlos causalmente con nuestros sentidos 1> %ara !oKe, sensaciones 5 percepciones son estados cogniti4os por()e proporcionan informacin $del c)erpo 5 del entorno), 5 por()e serian: a. portadores de rasgos cualitativos b. poseedoras de contenido conceptual c. facilitadores de conductas adaptativas 1? Deg1n la teora ad4er/ial, podemos e4itar post)lar la existencia de o/6etos internos en las sensaciones 5 percepciones, en tanto: a. el objeto puede ser reemplazado por acto" accin 4sonrisa por sonrer2 b. el objeto real puede ser reemplazado por objeto aparente 4veo un rbol por me parece un rbol c. el objeto interno puede ser reemplazado por objeto externo 4ver rojo por ser rojo2 1@ Deg1n la teora ca)sal, las experiencias percept)ales son ca)sadas por o/6etos externos- !os crticos de esta teora, considerando ()e las relaciones ca)sales son contingentes, dir.n ()e: a. es imposible !ue nunca percibamos objetos 8externos8 b. existe la posibilidad de !ue realmente nunca percibamos objetos 8externos8 c. no podramos tener todas las experiencias !ue de hecho tenemos en ausencia del mundo exterior A Deg1n !oKe, Les plateadoL se diferencia de Les plateadoGL por()e el primero es )n predicado ()e aplicado a cosas ) o/6etos significa algo as como Ltiene la disposicin a ca)sar datos sensoriales plateadosG en condicin de 4isin normal mientras ()e el seg)ndo es : a. el nombre !ue le damos al objeto interno del !ue somos conciente directamente b. un predicado !ue se aplica exclusivamente a datos sensoriales !ue causan tpicamente las cosas plateadas c. una propiedad disyuntiva !ue denotara una propiedad genuina de algunos objetos

%ER<E%<IVN $!'NE)
1 Respecto de la int)icin de sentido com1n de ()e c)ando )no est. perci/iendo )n o/6eto )no est. Ren contacto directo con el o/6etoR, la teora dis5)nti4a: a. la niega b. no la considera c. la respeta d. la refuta El concepto de R4isin ciegaR a. alude a los casos de sujetos es!uizofrnicos !ue en su delirio temen haber sido privados del sentido de la vista b. alude a los casos de sujetos ciegos !ue recuperan la vista c. alude al fenmeno de orientacin por ecolocacin de los murcilagos d. alude a los casos de sujetos !ue por da0os en las regiones corticales occipitales carecen de experiencia perceptual fenomnica aun cuando parecen procesar en alg*n nivel la informacin visual # 'tra diferencia entre los 6)icios 5 las experiencias percept)ales es ()e a. el contenido expresable en una proposicin sobre una experiencia perceptual es m s rico y menos puntual !ue la experiencia misma b. el contenido expresable en una proposicin sobre un juicio proporciona un inventario completo de los conceptos involucrados en dicho juicio c. el contenido expresable en una proposicin sobre una experiencia perceptual proporciona un inventario completo de los conceptos involucrados en dicho juicio d. los juicios no son expresables por medio de proposiciones & Deg1n el enfo()e comp)tacional )n sistema percept)al p)ede conce/irse como )n a. sistema de procesamiento de informacin !ue funciona de modo an logo a un ordenador b. ninguna de las otras respuestas es correcta c. sistema de procesamiento de informacin pero con potencial para el cmputo de experiencias subjetivas 4lo !ue lo hace levemente diferente de un ordenador2 d. sistema !ue a diferencia de los ordenadores se especializo en computar la informacin solo en el plano consciente 0 Entre ()e teoras act)ales de la percepcin plantea !oKe )na contraposicin: a. la teora causal vs. la teora ptica b. la teora ondulatoria vs. la teora disyuntiva c. la causal vs. la teora ondulatoria d. la teora causal vs. la teora disyuntiva 2 %ara !oKe: Rel tipo de conceptos en los c)ales se s)/s)men los o/6etos al tener experiencias percept)ales de esos o/6etos son conceptos c)5a posesin incorpora )n conocimiento implcito de como aparecen tales o/6etos caractersticamente a los sentidos, sea 4is)almente o t.ctilmente, o por 4a de otra modalidad sensorialR- En la anterior consideracin el a)tor se refiere a a. 9observaciones fenomnicas9 b. 9conceptos observacionales9 c. 9representaciones innatas9 d. 9!ualia9 > Desde el enfo()e ecolgico de la percepcin $de Gi/son) a. se sostiene !ue la percepcin supone principalmente procesos de tipo sint cticos

b. niega !ue el sistema visual humano este sincronizado para ad!uirir informacin del entorno c. se acepta !ue la percepcin es un proceso representacional indirecto d. se critica la idea de representacin y se sostiene !ue la percepcin es un proceso 9directo9 ? %ara la teora ca)sal de la percepcin, lo ()e 7ace ()e algo p)eda definirse como percepcin es a. !ue el objeto 4manzana2 y el contenido percibido 4imagen de la manzana2 sean causados por un estimulo fsico 4fotones y tomos2 b. !ue el objeto 4la manzana2 cause la percepcin 4de la imagen de la manzana2" el contenido percibido 4la imagen de la manzana2 se ajuste perfectamente al objeto 4la manzana2 y !ue por lo tanto" en presencia de un objeto diferente 4como un jarrn verde2 laN c. !ue el contenido percibido 4por ejemplo la imagen de una manzana2 se ajuste perfectamente al objeto 4la manzana2 d. !ue el objeto 4la manzana2 cause la percepcin 4una imagen determinada2" independientemente del nivel de ajuste entre el contenido percibido y el objeto @ Alg)nos filsofos 7an prop)esto ()e a. las cualidades perceptuales secundarias est n realmente en los objetos externos b. las cualidades perceptuales primarias est n realmente en los objetos externos c. no existen las cualidades perceptuales primarias d. las cualidades perceptuales secundarias no son verdaderamente percibidas 1A Respecto de 4er/os como RparecerR 5 RsentirR !oKe sostiene ()e poseen al menos dos )sos caractersticos: a. uno epistemolgico y otro fenomnico b. uno ortodoxo y otro heterodoxo c. uno inmanente y otro trascendental d. uno metafsico y otro cientfico 11 !os ad4er/ios ()e descri/en el modo en ()e las cosas nos parecen ser a. poseen 9significados primitivo9 b. ad!uieren su significado por aprendizaje asociativo c. ad!uieren su significado mediante definiciones ostensivas d. no poseen significado 1 Desde la teora ad4er/ial de las sensaciones, se sostiene ()e )n en)nciado como Rtengo )n dolor f)erte en la espaldaR a. es gramaticalmente parecida a 9me duele la muela9 pero no posee el mismo estatus lgico b. es gramaticalmente diferente a 9me duele la muela9 y posee diferente estatus lgico c. es gramaticalmente diferente a 9tengo una bicicleta vieja en el cobertor9 pero tiene el mismo estatus lgico d. es gramaticalmente parecida a 9tengo una bicicleta vieja en el cobertor9 pero no tiene el mismo estatus lgico 1# !o ()e se les reproc7a a los Rrealistas directosR es a. niegan los !ualia pero en su lugar pretenden introducir la nocin de 9hecho consciente9 b. no dan cuenta de cmo se constituye el 9espacio9 en !ue sit*an a los objetos internos c. niegan los datos sensoriales sin ofrecer nada en su lugar d. !ue no tienen una buena teora electrofisiolgica respecto al modo en !ue estmulos fsicos se traducen al nivel electrofisiolgico

1& El pro/lema de )no de los arg)mentos presentado por los tericos de los datos sensoriales para defender s) tesis es ()e a. supone !ue la percepcin de ciertas caractersticas o cualidades implica la existencia de un objeto !ue posee realmente ciertas propiedades !ue hacen posibles a las primeras b. supone !ue la percepcin de ciertas caractersticas o cualidades implica necesariamente la existencia de experiencias subjetivas c. supone !ue la percepcin de ciertas caractersticas o cualidades no depende de la participacin de nuestro sistema nervioso d. supone !ue la percepcin de ciertas caractersticas o cualidades depende de sensaciones en niveles subumbral 10 Deg1n !oKe los Ren)nciados percept)alesR son partic)lares p)es: a. se asemejan a las actitudes proposicionales b. se asemejan tanto a los enunciados de sensacin como a las actitudes proposicionales c. se asemejan a los enunciados de sensacin d. se asemejan a los enunciados conductuales 12 Ena de las criticas RontolgicasR al s)p)esto de la existencia de o/6etos internos es ()e a. parecen ser objetos fsicos sutiles explicables en trminos de potenciales elctricos a largo plazo b. no parece !ue sean objetos fsicos ni !ue ocupen un lugar en el espacio aun!ue posean ciertas caractersticas espaciales c. parecen ocupar un lugar dentro de lo !ue podramos llamar el espacio mental d. parecen ser objetos fsicos aun!ue no ocupen lugar en el espacio 1> Eno de los pro/lemas centrales en la filosofa de la mente a/ocada a tra/a6ar los pro/lemas de la percepcin 7a sido a. establecer si la percepcin constituye un autentico fenmeno mental b. decidir sobre si debiramos reificar a los !ualia consider ndolos especies de objetos internos c. determinar si los !ualia pueden o no ser reducidos a una experiencia b sica fundamental d. establecer si la percepcin es una forma de conducta encubierta

%ER<E%<IVN $DEAR!E)
1) De ac)erdo con Dearle la ma5ora de los filsofos cree ()e el realismo ingen)o es falso- As, se sostiene )na posicin ()e post)la la existencia de Rdatos sensorialesR- En este sentido, estos filsofos defienden ()e a2 )enemos acceso directo a los 9objetos materiales9 b2 )enemos acceso directo slo a nuestras experiencias internas c2 )enemos acceso directo tanto a nuestras experiencias internas como a los objetos materiales ) %ara criticar el realismo ingen)o se 7a prop)esto Rel arg)mento de la cienciaR- %ara Dearle, este arg)mento es )na forma de la falacia gen8tica por()e: a2 +upone !ue podemos refutar una creencia si podemos mostrar !ue las causas de una creencia son insuficientes para probar su verdad b2 +upone !ue podemos refutar una creencia si podemos mostrar !ue no tiene condiciones bajo las cuales puede ser verdadera o falsa c2 +upone !ue podemos refutar una creencia al apelar a sus condiciones empricas #) 'tro arg)mento ()e se 7a prop)esto para Rref)tarR al realismo ingen)o es el de la il)sinEste arg)mento s)pone, entre otras cosas, ()e: a2 los casos de experiencias alucinatorias y los casos de experiencias verdicas no poseen un car cter cualitativo distintivo ni particular b2 los casos de experiencias alucinatorias y los casos de experiencias verdicas tienen distinto car cter cualitativo c2 los casos de experiencias alucinatorias y los casos de experiencias verdicas tienen el mismo car cter cualitativo &) Dearle propone )n arg)mento trascendental para defender el realismo ingen)o- En este arg)mento intenta defender ()e dic7o realismo ingen)o es )na Rcondicin de posi/ilidadR para ()e se de el fenmeno de a2 la comunicacin entre los seres humanos b2 la generacin de conocimiento cientfico por parte de los seres humanos c2 la socializacin entre los seres humanos

%ENDAFIENP' \ !ENGEAXE $!'NE)


1 Al parecer de !oKe )n re()isito para el )so de )n leng)a6e es a. !ue uno posea algunas nociones explicitas relativas a las derivaciones de los verbos b. !ue uno cuente con alguna forma de experiencia subjetiva previa !ue comunicar c. !ue uno pueda concebir a sus congneres como seres !ue tienen pensamientos !ue comunicar d. !ue uno tenga ciertos conocimientos explcitos sobre la gram tica del sujeto y el predicado Ena esfera de )n relo6 ()e marca la 7ora mediante ag)6as, expresando el paso del tiempo en f)ncin de las distancias de dic7as ag)6as respecto de )n p)nto de referencia, es )na forma de representacin a. dialgica b. lgica c. analgica d. digital # <on el concepto de Rprod)cti4idad del leng)a6eR se al)de a a. su capacidad para producir un n*mero indefinido de 9textos9 posibles b. su capacidad para producir diferentes sistemas gr ficos por medio de los cuales representar sus sonidos c. su capacidad para generar un n*mero indefinidamente grande de oraciones a parir de un vocabulario limitado d. su capacidad para articular un n*mero infinito de sonidos para con ellos construir una lengua & %ara !oKe los sistemas de com)nicacin intraespecfica de ciertos animales como los monos: a. constituye un autentico lenguaje pues se ha demostrado !ue en sus mensajes se hallan implicadas ciertas formas b sicas de sintaxis b. constituye un autentico lenguaje pues se ha demostrado !ue al igual !ue sucede con los sordos" los sistemas !ue emplean gestos no implican sintaxis c. no constituyen un autentico lenguaje pues no tiene estructura sint ctica ni goza de 9productividad9 d. no constituye un autentico lenguaje pues no supone procesos de verbalizacin 0 Respecto de las Rconstr)cciones representacionales 7)manasR como los mapas o los di/)6os !oKe sostiene ()e: a. no poseen intencionalidad derivada pero poseen contenido intencional b. poseen una forma de intencionalidad derivada c. poseen intencionalidad intrnseca d. no poseen intencionalidad 2 El t8rmino Rleng)a6e del pensamientoR o RmentalesR al)de a a. el solilo!uio silencioso !ue nos dedicamos cuando debemos pensar el modo de resolver un problema complicado b. el habla interior !ue empleamos cuando pensamos o hacemos juicios c. una forma no manifiesta !ue expresa nuestros pensamientos en el mismo lenguaje !ue hablamos d. un conjunto de representaciones o conceptos primitivos !ue se articulan a travs de una serie de reglas gramaticales mediante de modo parecido a lo !ue ocurre con las lenguas naturales y !ue describe al lenguaje mediante el cual pensamos > De ac)erdo con la 7iptesis de N7orf $citado en !oKe) las categoras gramaticales de las leng)as indoe)ropeas

a. parecen corresponderse con las categoras ontolgicas de la metafsica occidental b. parecen corresponderse con las gramaticales del mentals c. parecen corresponderse con las de la lengua Bopi d. parecen responder a un sistema de ense0anza de la lengua en las escuelas ? Deg1n !oKe, Donal Da4ison arg)menta ()e para poseer estados de creencia a. uno debe tener la capacidad de conformar oraciones con la forma de una actitud proposicional b. basta con !ue uno tenga cierta experiencia socioafectiva elemental c. uno debe poseer el concepto de creencia d. uno no necesita poseer conceptos previos @ De ac)erdo con !oKe a. no se ha demostrado !ue la forma de representacin mental exclusivamente analgica o digital b. se ha demostrado !ue la forma de representacin mental es dialgica c. se ha demostrado !ue la forma de representacin mental es analgica d. se ha demostrado !ue la forma de representacin mental es digital 1A !a intencionalidad original de )na representacin consiste en: a. !ue una cosa representa alguna otra b. la intervencin de un intrprete c. la capacidad para dar una informacin sobre el mundo 11 !a prod)cti4idad del leng)a6e 7)mano se /asa en: +eleccione una respuesta. a. el vocabulario b. !ue haya muchos usuarios de ese lenguaje c. la sintaxis d. !ue sirva para la comunicacin sea

%ENDAFIENP' \ !ENGEAXE $!'NE) 9


1) <onstr)cciones representacionales 7)manas, como los di/)6os o los mapas, tienen: a. estados mentales b. intencionalidad original c. intencionalidad derivada d. ninguna intencionalidad ) El leng)a6e de la mente si existe, es )n leng)a6e con en el c)al pensamos pero, es: a. idntico solamente a la lengua materna b. diferente del lenguaje en el cual hablamos c. idntico a cual!uier lenguaje en el cual podamos hablar d. el mismo del habla interior #) !a intencionalidad de )na representacin consiste en: a. la intervencin de un intrprete b. su capacidad para dar una informacin sobre el mundo c. su contenido conceptual d. una cosa representa alguna otra e. su utilidad &) !a prod)cti4idad del leng)a6e se /asa en: a. el vocabulario b. la sintaxis c. !ue sirva para la comunicacin d. !ue haya muchos usuarios de ese lenguaje

0) !os relati4istas ling]sticos sostienen: a. la estructura ontolgica del mundo es independiente de los es!uemas conceptuales b. la estructura del mundo impone los es!uemas conceptuales a los hablantes c. un realismo respecto a la estructura ontolgica del mundo d. el es!uema conceptual impone al hablante una estructura ontolgica particular 2) El conocimiento propiamente dic7o est. 4inc)lado insepara/lemente a a. capacidad de utilizar conceptos b. capacidad de tener sensaciones c. capacidad de formular oraciones d. capacidad de observacin e. capacidad de percibir >) Ba/lar de LrepresentacinL en el contexto de los estados cogniti4os, incl)5endo pensamientos 5 creencias $()e son a la 4e" estados mentales 5 estados representacionales), no permite discriminar entre: a. teora y pr ctica b. sensacin y percepcin c. conceptos tericos y conceptos observacionales d. realidad y alucinacin e. contenido conceptual y contenido no3conceptual ?) !a intencionalidad deri4ada se de/e a ()e se re()iere: a. contenido conceptual b. contenido no conceptual c. la accin de un intrprete d. construccin humana @) !a seme6an"a ()e existe entre el leng)a6e del pensamiento 5 leng)a6es nat)rales, es )na seme6an"a: a. de vocabulario b. de contenido conceptual c. informativa d. estructural 1A) Deg1n !oKe los diagramas 5 mapas son e6emplos de: a. representaciones no mentales b. representaciones mentales c. no representaciones d. conceptos e. representaciones digitales 11) %ara !'NE, atri/)ir a )n animal )n estado de actit)d proposicional como la creencia de ()e en s) plato 7a5 comida, in4ol)cra ,. Los conceptos involucrados en tal creencia 4comida" plato" etc."2 E. ,tribuirle la inteligencia para comportarse de modo adaptativo en relacin con su alimento. C. ,tribuirle un contenido no3conceptual perceptual. 1 ) %ara !oKe: ,. El concepto de inteligencia es m s amplio !ue el de pensamiento" abarcando las conductas bien adaptadas a las necesidades del animal. E. El concepto de inteligencia es e!uivalente al de pensamiento. C. El concepto de inteligencia en m s estrecho !ue el de pensamiento" pues exige !ue la criatura no solo tenga pensamientos" sino !ue los emplee en razonamientos. 1#) %ara !oKe poseer pensamientos concept)ales es: ,. Condicin necesaria para la posesin de lenguaje. E. -o es condicin ni necesaria ni suficiente para poseer lenguaje. C. Condicin necesaria y suficiente para la posesin de lenguaje.

1&) %ara !oKe, el 4oca/)lario ()e forma parte de n)estro leng)a6e nat)ral: ,. Condiciona el espectro de conceptos !ue podemos tener" solo en a!uellos casos en los !ue no tengamos modo de describir en nuestro lenguaje los trminos del otro vocabulario para los cuales no hay un trmino e!uivalente en nuestro vocabulario. E. Condiciona el espectro de conceptos !ue podemos tener" de modo tal !ue" por ejemplo" las personas !ue hablan lenguajes !ue contienen trminos !ue distinguen mayor cantidad de matices de color" poseen un espectro de conceptos m s ricos con respecto a ese color. C. -o condiciona el espectro de de conceptos !ue podemos tener" pues podemos emplear nuestro lenguaje para formar una expresin compleja !ue describa las palabras de la otra lengua para las cuales no hay un trmino e!uivalente en nuestro vocabulario. 10) %ara <7omsQ5 los seres 7)manos poseeramos conocimiento innato de: ,. Ciertos rasgos sint cticos generales comunes a todos los lenguajes. E. -o tendramos ning*n conocimiento innato: los lenguajes naturales son aprendidos. C. Las reglas sint ctica de todos los lenguajes en detalle. '. Ciertos conceptos sem nticos fundamentales presentes en todos los lenguajes.

%ENDAFIENP' \ !ENGEAXE $!'NE) 9 #


1) Deg1n !oKe: a2 Los mapas" las fotografas y otras representaciones pictricas b2 El lenguaje y los n*meros c2 Los contenidos mentales d2 El lenguaje" los mapas y las fotografas - Ena representacin es analgica c)ando: $4arias opciones) a2 /uestra un parecido con el objeto representado b2 'escribe el objeto representado c2 +e compone de partes discretas d2 Bay una relacin de continuidad entre sus componentes. #- !as concepciones seg1n las c)ales n)estro pensamiento se compone de 3im.genes mentales:: a2 )ienen dificultades para explicar la especificidad 4o singularidad2 !ue muestran algunos estados mentales. b2 +on incompatibles con la nocin de modelos mentales defendida por &erner c2 -o pueden dar cuenta de ciertos reportes de sentido com*n" basados en la introspeccin. &- !a 7iptesis del leng)a6e del pensamiento $mental8s): $4arias opciones) a2 es una hiptesis acerca del modo en !ue hacemos introspeccin b2 +e basa en un modelo de representacin digital c2 +e basa en un modelo de representacin analgica d2 Es una hiptesis acerca de las propiedades estructurales del pensamiento 0) seg1n la 7iptesis del leng)a6e del pensamiento, la prod)cti4idad $4arias opciones) a2 es una propiedad sint ctica

b2 es un rasgo distintivo del lenguaje natural c2 es un rasgo compartido por el lenguaje natural y el pensamiento d2 es una propiedad sem ntica.

%ENDAFIENP' \ !ENGEAXE $!'NE) S &


1) !a intencionalidad original de )na representacin consiste en: a. la intervencin de un intrprete b. !ue una cosa representa alguna otra c. la capacidad para dar una informacin sobre el mundo ) Deg1n !oKe, las constr)cciones representacionales 7)manas como los mapas tienen intencionalidad originalDerdadero .also

#) Ba/lar de LrepresentacinL en el contexto de los estados cogniti4os, incl)5endo pensamientos 5 creencias $()e son a la 4e" estados mentales 5 estados representacionales), no permite discriminar entre: a. teora y pr ctica b. contenido conceptual y contenido no3conceptual c. sensacin y percepcin d. conceptos tericos y conceptos observacionales e. realidad y alucinacin &) !a prod)cti4idad del leng)a6e 7)mano se /asa en: a. !ue sirva para la comunicacin b. !ue haya muchos usuarios de ese lenguaje c. la sintaxis d. el vocabulario 0) !os relati4istas ling]sticos sostienen: a. la estructura ontolgica del mundo es independiente de los es!uemas conceptuales b. la estructura del mundo impone los es!uemas conceptuales a los hablantes c. un realismo respecto a la estructura ontolgica del mundo d. el es!uema conceptual impone al hablante una estructura ontolgica particular 2) !as representaciones mentales: a. tienen intencionalidad derivada b. no tienen elementos de representacin analgica c. no tienen elementos de representacin digital d. tiene intencionalidad original >) Ena representacin es analgica c)ando 7a5 )n parecido entre la representacin 5 lo representadoDerdadero .also

?) !a intencionalidad de )na representacin s) capacidad para dar )na informacin so/re el m)ndoDerdadero .also @) El conocimiento propiamente dic7o est. 4inc)lado insepara/lemente a: a. capacidad de percibir b. capacidad de observacin c. capacidad de tener sensaciones d. capacidad de utilizar conceptos 1A) El leng)a6e de la mente 9si existe9 es )n leng)a6e con el c)al pensamos, pero es: a. idntico solamente a la lengua materna b. el mismo del habla interior c. diferente del lenguaje en el cual hablamos d. idntico a cual!uier lenguaje en el cual podamos hablar

RA<I'NA!IDAD BEFANA E IA $!'NE \ <BER<B!AND)


1 Dentro de la inteligencia artificial, )n sistema formal consiste en: a. un conjunto de operaciones intrnsecamente inteligentes y capaces de generar actos creativos b. un conjunto de frmulas y de reglas de transformacin para manejarlas c. un conjunto de datos codificados en un lenguaje natural d. un conjunto de normas heursticas En inteligencia artificial, al m8todo ()e l)ego de descri/ir )n pro/lema /)sca )na sol)cin a partir de )n .r/ol de /1s()eda $()e a/arca todas las posi/ilidades pertinentes, 5 s)pone )n an.lisis ex7a)sti4o de las ramas del .r/ol), se lo denomina: a. mtodo del rbol b. mtodo de fuerza bruta c. mtodo de la heurstica d. mtodo de la decisin exhaustiva # !a Linteligencia artificial d8/ilL sostiene ()e: a. Es posible dise0ar programas inform ticos !ue modelen o simulen diversos aspectos de la conducta humana inteligente b. Existe la posibilidad de desarrollar una m !uina capaz de realizar algunas 4aun!ue no todas2 de las actividades inteligentes tpicamente humanas c. -o se puede dise0ar m !uinas capaces de llevar a cabo adecuadamente distintas actividades !ue realizan los seres humanos cuando aplican su 8sentido com*n8 en la vida diaria & !oKe compara la Rlgica mentalR con los Rmodelos mentalesR, en tanto modos en ()e ra"onamos ded)cti4amente- Deg1n la primera, )samos nat)ralmente las reglas de ra"onamiento ded)cti4oH para los seg)ndos, a. solo manejamos inferencias indirectas y reducciones al absurdo b. no aplicamos reglas formales de inferencias c. manejamos de manera t cita o implcita todas las reglas como el modus tollens 0 !oKe seala ()e la %r)e/a de P)ring s)rge de la idea de ()e el criterio ()e tenemos para 6)"gar a otro ser 7)mano como inteligente se /asa en: a. la capacidad de di logo !ue podra tener un computador b. la conducta observable verbal y no verbal c. el poder de c lculo computacional d. el ingenio e intuicin de las personas 2 %ara <7)rc7land, respecto de la inteligencia 5 la conciencia p)ede decirse ()e: a. !ue se encuentran en grados diferentes en el hombre" las m !uinas y todo el reino animal b. !ue no son reproducibles en una m !uina c. se encuentran en grados semejantes en las m !uinas y los animales > De p)ede descri/ir el Lpro/lema del marcoL como: a. El problema !ue encuentran los dise0adores de $, para dise0ar un programa !ue incorpore los conocimientos generales del ser humano ordinario y especifi!ue adem s el modo de aplicar el mismo a las circunstancias pertinentes b. La dificultad !ue experimental los dise0adores de $, para enmarcar el conjunto de reglas !ue permitan a un ordenador realizar tareas especficas como jugar adecuadamente al ajedrez. c. (na dificultad !ue encuentran los defensores del conexionismo para dise0ar programas capaces de llevar a cabo programas inteligentes capaces de articular una amplia variedad de actividades cognitivas ? Deg1n !oKe, las in4estigaciones empricas lle4adas a ca/o por los psiclogos para e4al)ar la racionalidad 7)mana tienen el defecto de ()e slo e4al1an la capacidad de

a. razonamiento pr ctico b. razonamiento terico c. razonamiento deductivo d. razonamiento inductivo @ Deg1n <7)rc7land, )na caracterstica del procesamiento de informacin en las criat)ras /iolgicas, ()e no es compartida con la ma5or parte de los comp)tadores artificiales, es el procesamiento: a. en serie b. en paralelo c. digital d. analtico 1A %ara <7)rc7land, )na de las .reas ()e m.s dific)ltad presenta para la ela/oracin de programas inteligentes en el plano de la inteligencia artificial es el .rea de: a. la resolucin de problemas logartmicos de alta complejidad b. la comprensin de problemas algortmicos complejos c. la resolucin de problemas lgicos de primer orden d. la produccin y comprensin de un lenguaje natural 11 Deg1n !oKe, el experimento mental de la Ba/itacin <7ina prop)esto por Dearle, m)estra ()e la %r)e/a de P)ring: a. slo implica la sem ntica en el manejo de smbolos b. es un buen criterio de comprensin del chino c. slo implica la sintaxis en el manejo de smbolos d. es una buena definicin de la inteligencia e. slo implica inteligencia computacional 1 Deg1n !oKe, el pre6)icio de empare6amiento ()e se prod)ce al resol4er la tarea de seleccin de Nason no permite 6)stificar el arg)mento de ()e los seres 7)manos no ra"onamos lgicamente- D) rec7a"o se de/e a ()e: a. nada justifica !ue el razonamiento deductivo terico sea el modo correcto de razonar b. no hay motivos para considerar !ue la tarea de seleccin de %ason sea un buen experimento de razonamiento c. los seres humanos slo razonan lgicamente si han aprendido a usar la lgica 1# Deg1n <7)rc7land, los sistemas artificiales no p)eden mane6ar leng)a6es nat)rales, de/ido a ()e nosotros mismo no sa/emos cmo ad()irimos todo el conocimiento general so/re el m)ndo ()e necesitamos para )sar )n leng)a6ea. Derdadero b. .also 1& El L%ro/lema del FarcoL, mencionado por !oKe como )n reto para los defensores de la IA, refiere a la imposi/ilidad de escri/ir )n programa ()e no slo incorpore los conocimientos generales del ser 7)mano sino ()e adem.s: a. especifi!ue el modo en !ue dicho conocimiento debe aplicarse adecuadamente a las circunstancias pertinentes b. pase exitosamente el experimento de la Babitacin China c. especifi!ue el modo en !ue dicho conocimiento debe aplicarse adecuadamente a las pruebas cl sicas de inteligencia d. pase exitosamente la &rueba de )uring 10 En inteligencia artificial, al m8todo ()e consiste en programar a la comp)tadora con estrategias de resol)cin de pro/lemas ()e le permitan tomar )na decisin con alta pro/a/ilidad de ser correcta, se la denomina: a. mtodo del rbol b. mtodo heurstico c. mtodo de la decisin exhaustiva d. mtodo de fuerza bruta

12 !a caracterstica de las representaciones 4is)ales ()e 7ace difcil el correcto diseo de procesadores no /iolgicos de esta clase de informacin es ()e las mismas: a. presentan una gama extremadamente amplia de colores b. son extremadamente ambiguas c. no pueden codificarse pues la energa no es transformable a un sistema de pulsos elctricos 1> !a crtica principal al Pest de P)ring ()e se desprende del experimento de mental de la 7a/itacin c7ina consiste en sostener ()e: a. ,l igual !ue ocurre en el caso de la habitacin china" los hombres no estaramos posibilitados de analizar las interacciones de la m !uina con su ambiente circundante" ya !ue slo contaramos con sus 8respuestas8 a nuestras 8preguntas8 b. 'e manera semejante al hombre encerrado en la habitacin china" la m !uina !ue pasa el )est de )uring carecera de un dominio sint ctico adecuado del lenguaje c. 'e manera an loga al hombre encerrado en la habitacin china" una m !uina podra ser capaz de dar respuestas correctas a nuestras preguntas sin comprender el lenguaje !ue utiliza 1? !oKe considera ()e el <onexionismo, al ig)al ()e la IA <ogniti4ista, no p)ede crear ni sim)lar RinteligenciaR por()e: a. solo posee sem ntica y recurre a capas ocultas de procesamiento b. simula procesos en paralelo pero de manera distribuida c. slo posee sintaxis y explica a nivel funcional 1@ !oKe disting)e dos nociones de racionalidad- <ada )na de estas nociones prop)estas por !oKe podra relacionarse con )na capacidad especfica, a sa/er: a. la capacidad de razonar deductivamente y la capacidad de razonar pr cticamente b. la capacidad de razonar tericamente y la capacidad de adecuar acciones a objetivos" al entorno y a los medios disponibles c. la capacidad adecuar acciones a objetivos" al entorno y a los medios disponibles y la capacidad de ser racional d. la capacidad de razonar tericamente y la capacidad de razonar deductiva o inductivamente A %ara <7)rc7land, el aprendi"a6e, en sentido no conser4ador: a. es solamente la posibilidad de representar la informacin en una categora de es!uemas y categoras provista por el programa original b. solo se da en la m !uina o computadora c. es tan solo el almacenamiento de informacin ad!uirida d. solo se da en su m xima expresin cuando un procesador puede generar categoras completamente nuevas

RA<I'NA!IDAD BEFANA $!'NE)


1) Pradicionalmente la racionalidad f)e conce/ida como )na capacidad ()e slo poseen los seres 7)manos- Deg1n !oKe esta idea f)e c)estionada por: a2 El hecho de !ue los animales 4no humanos2 demostraron su capacidad para realizar razonamientos de tipo terico. b2 El hecho de !ue los seres humanos demostraron no pensar tan racionalmente como se crey c2 El hecho de !ue los animales 4no humanos2 demostraron su capacidad para realizar razonamientos de tipo pr ctico ) El ra"onamiento p)ede ser de tipo pr.ctico o de tipo terico- !o ()e se entiende por ra"onamiento de tipo prctico es: a2 (n razonamiento !ue prioriza la resolucin concreta de las situaciones en desmedro de los valores o ideales !ue alguien pueda tener. b2 (n razonamiento !ue permite calcular cu l es el medio m s adecuado para conseguir un fin determinado !ue alguien pueda tener. c2 (n razonamiento !ue" en la resolucin de una situacin problem tica" prioriza el beneficio inmediato !ue alguien pueda tener. #) En ra"onamiento de tipo ind)cti4o es definido del sig)iente modo: a2 est compuesto por premisas y conclusin" donde las premisas son particulares y la conclusin es general. Es decir" un razonamiento inductivo es el paso de lo particular a lo general. b2 est compuesto por premisas y conclusin" donde el paso inferencial de las premisas a la conclusin es 4altamente2 probable" pero contingente. c2 est compuesto por premisas y conclusin" donde el paso inferencial de las premisas a la conclusin es necesario. &) En ra"onamiento de tipo ded)cti4o es definido del sig)iente modo: a2 est compuesto por premisas y conclusin" donde las premisas son generales y la conclusin es particular. Es decir" un razonamiento deductivo es el paso de lo general a lo particular. b2 est compuesto por premisas y conclusin" donde el paso inferencial a la conclusin tiene un gran apoyo por parte de las premisas. c2 est compuesto por premisas y conclusin" donde el paso inferencial de las premisas a la conclusin es necesario. 0) El experimento de tipo emprico reali"ado por Nason, denominado 3la tarea de seleccin de Nason:, permite ()e los psiclogos lleg)en a la sig)iente concl)sin: a2 Las personas no logran resolver de manera correcta el razonamiento deductivo implicado en la tarea de seleccin de %ason por!ue son irracionales. b2 Las personas no logran resolver de manera correcta el razonamiento deductivo implicado en la tarea de seleccin de %ason por!ue no conocen cu les son las formas !ue un razonamiento deductivo debe respetar. c2 Las personas no logran resolver de manera correcta el razonamiento deductivo implicado en la tarea de seleccin de %ason por!ue utilizan JprejuiciosK para razonar. 2) !a tarea de Nason implica )n pro/lema de tipo: a2 Emprico" pues lo importante es saber cmo de hecho resuelven las personas esta tarea. b2 -ormativo" pues lo importante es saber cmo deben resolver las personas esta tarea. c2 Lgico" pues lo importante es saber !u es la lgica deductiva. >) El experimento reali"ado por &a'neman y ( ers)y m)estra ()e las personas sostiene ()e la pro/a/ilidad de ()e )n terremoto ca)se 4ctimas fatales en <alifornia ED FA\'R a la pro/a/ilidad de ()e ca)se 4ctimas fatales en Am8rica del Norte, p)es en <alifornia oc)rren m)c7os

terremotos 5 los terremotos generan 4ctimas fatales- Din em/argo, esta concl)sin N' es correcta, por()e <alifornia c)enta con )n excelente sistema de alerta para estos casos- Este experimento p)ede ser considerado como )n /)en e6emplo de: a2 Los errores !ue comenten las personas al razonar deductivamente. b2 Los errores !ue comenten las personas al razonar inductivamente. c2 Los errores !ue comenten las personas al razonar utilizando silogismos. ?) A la l)" de los e6emplos de la tarea de Nason 5 de la falacia de la proporcin /.sica, los psiclogos sostiene ()e es posi/le o/tener datos so/re los pre6)icios presentes en los ra"onamientos 5 so/re los modos de ra"onar- En este sentido sostienen ()e las personas )tili"an )n sistema de lgica mental, el c)al consiste en: a2 $maginar cada una de las premisas del razonamiento y realizar la inferencia necesaria para llegar a la conclusin correcta. b2 (tilizar un sistema de lgica !ue cada uno de nosotros construimos en nuestra mente. c2 (tilizar las reglas de la lgica formal para realizar las inferencias necesarias para pasar de las premisas a la conclusin. @) !os psiclogos sostienen ()e )n modo de ra"onar, alternati4o a los sistemas de lgica mental, es el modelo mental- <)ando )na persona )tili"a modelos mentales: a2 +iguen un modelo ejemplar en su mente !ue les muestra cu l es la conclusin correcta. b2 Construye JmundosK !ue representan a las premisas y" al combinar estos JmundosK" las personas eliminan algunos por inconsistentes y descubren cu l es la conclusin correcta. c2 (tiliza las reglas formales lgicas como modelos para saber cu l es la conclusin correcta. 1A) !a racionalidad p)ede ser entendida como: a2 )odas las opciones son correctas b2 La capacidad de realizar buenos razonamientos de tipo pr ctico o terico c2 La capacidad de realizar buenos razonamientos de tipo deductivo o inductivo d2 La capacidad de adaptarse a su medio fsico y social

INPE!IGEN<IA ARPI+I<IA! $!'NE)


1) !a Inteligencia Artificial $IA) es: a2 (na teora filosfica acerca del pensamiento humano propuesta primeramente por #. Leibniz. b2 (n conjunto de disciplinas cientficas centradas en el uso de ordenadores y la elaboracin de programas inform ticos para el estudio de la inteligencia c2 -inguna de las anteriores. d2 El nombre de un conocido argumento de O. +earle seg*n el cual los ordenadores pueden considerarse inteligentes. ) !a 7iptesis central ()e est. detr.s de la IA es ()e: a2 -inguna de las anteriores. b2 &uede elaborarse un programa inform tico general capaz de desempe0ar toda actividad inteligente. c2 )odos los procesos !ue subyacen a la inteligencia consciente son procedimientos computacionales. d2 La verdadera inteligencia es a!uella propia de los ordenadores" dado su gran poder de c lculo y de almacenamiento de informacin. #) !a diferencia entre 3IA f)erte: e 3IA d8/il: consiste en ()e: a2 -inguna de las anteriores b2 /ientras para la primera podr simularse la inteligencia en general" para la segunda slo podr n simularse algunos de sus aspectos. c2 /ientras para la primera podr simularse la inteligencia en general" para la segunda slo podr n simularse algunos de sus aspectos. d2 /ientras para la segunda slo pueda hablarse de simulacin" para la primera un ordenador adecuadamente programado efectivamente tendra estados y procesos mentales. &) !a pr)e/a de P)ring es: a2 (na prueba centrada en la conducta verbal para evaluar las habilidades cognitivas de personas sin formacin en lgica. b2 (n experimento mental !ue demuestra !ue los ordenadores digitales son capaces de inteligencia genuina. c2 (na prueba !ue supera todo prejuicio antropocntrico en la medida en !ue contempla la conducta no verbal. d2 -inguna de las anteriores 0) Entre las 4enta6as del procesamiento en procesamiento serial est.n: a2 &recisin y buen desempe0o en problemas lgicos. b2 Lapidez" robustez y flexibilidad. c2 /ayor simplicidad y facilidad de programacin d2 -o existen ventajas puntuales. paralelo respecto del

RA<I'NA!IDAD E INPE!IGEN<IA ARPI+I<IA!


1) %ara <7)rc7land, respecto de la inteligencia 5 la conciencia p)ede decirse ()e: a. !ue no son reproducibles en una ma!uina b. se encuentran en grados semejantes en las ma!uinas y los animales c. !ue se encuentran en grados diferentes en el hombre" las ma!uinas y todo el reino animal ) Deg1n !oKe, las in4estigaciones empricas lle4adas a ca/o por los psiclogos para e4al)ar la racionalidad tienen como limitacin ()e slo e4al1an la capacidad de ra"onamiento ded)cti4oDerdadero .also

#) De p)ede descri/ir el %ro/lema del Farco como: a. El problema !ue encuentran los dise0adores de $, para dise0ar un programa !ue incorpore los conocimientos generales del ser humano ordinario y especifi!ue adem s el modo de aplicar el mismo a las circunstancias pertinentes b. (na dificultad !ue encuentran los defensores del conexionismo para dise0ar programas capaces de llevar a cabo programas inteligentes capaces de articular una amplia variedad de actividades cognitivas c. La dificultad !ue experimental los dise0adores de $, para enmarcar el conjunto de reglas !ue permitan a un ordenador realizar tareas especificas como jugar adecuadamente al ajedrez &) Deg1n !oKe, ser inteligente re()iere poseer conocimientos, capacidad de ampliarlos 5 considerar ()e las acti4idades cotidianas son acti4idades a)tocontenidas sin reglas fi6asDerdadero .also

0) El enfo()e de la !gica Fental de la racionalidad 7)mana sostiene ()e: a. Los seres humanos utilizan de manera natural ciertas reglas formales para realizar razonamientos deductivos" aun cuando no posean formacin en lgica formal b. Los seres humanos utilizan de manera natural cierta forma de lgica inductiva en sus razonamientos naturales" pero a!uellos !ue han recibido formacin en lgica deductiva privilegian esta *ltima c. Los seres humanos utilizan innatamente una combinacin de reglas lgicas y modelos mentales para realizar razonamientos en la vida cotidiana 2) !a crtica principal al Pest de P)ring, ()e se desprende del experimento mental de la Ba/itacin <7ina, consiste en sostener ()e: a. 'e manera an loga al hombre encerrado en la habitacin china" una ma!uina podra ser capaz de dar respuestas correctas a nuestras preguntas sin comprender el lenguaje !ue utiliza b. 'e manera semejante al hombre encerrado en la habitacin china" la ma!uina !ue pasa el )est de )uring carecera de un dominio sint ctico adecuado del lenguaje c. ,l igual !ue ocurre en el caso de la habitacin china" los hombres no estaramos posibilitados de analizar las interacciones de la ma!uina con su ambiente circundante" ya !ue solo contaramos con sus 8respuestas8 a nuestras 8preguntas8 >) Dentro de la inteligencia artificial, )n sistema formal consiste en: a. un conjunto de formulas y de reglas de transformacin para manejarlas b. un conjunto de datos codificados en un lenguaje natural c. un conjunto de normas heursticas d. un conjunto de operaciones intrnsecamente inteligentes y capaces de generar actos creativos

?) El ra"onamiento pr.ctico tiene como fin, seg1n !oKe: a. establecer la verdad b. el conocimiento cotidiano c. la accin exitosa d. establecer la diferencia entre razonamientos deductivos e inductivos

@) Deg1n !oKe, la tarea de seleccin de Nason constit)5e a. un problema de razonamiento practico b. un problema de razonamiento deductivo c. un problema de razonamiento terico d. un problema con entidades abstractas 1A) Deg1n <7)rc7land, los sistemas artificiales no p)eden mane6ar leng)a6es nat)rales, de/ido a ()e nosotros mismo no sa/emos cmo ad()irimos todo el conocimiento general so/re el m)ndo ()e necesitamos para )sar )n leng)a6eDerdadero .also

A<<IVN, !IURE A!UEDRO' E IN<'N<IENPE $DEAR!E Z@A \ ZA&)


1) Dearle afirma ()e la nocin de inconciente es )na de las concepciones m.s conf)sas e insensatas de la 4ida intelect)al moderna- %or ello, a. propone el principio de la conexin" basado en conceptos neurobiolgicos b. propone descartar el concepto de inconciente profundo y aceptar exclusivamente el concepto de preconciente c. propone abandonar el concepto de inconciente dentro del mbito de la ciencia" por irrelevante ) Dearle afirma ()e es posi/le explicar la estr)ct)ra de la cond)cta 7)mana rec)rriendo a )na serie de principios ()e expli()en los aspectos fsicos de las accionesDerdadero .also

#) Dearle afirma ()e si el componente mental de la accin tiene 8xito, ca)sa el componente fsico- A esta forma de ca)sacin se la llama Rca)sacin intencionalRDerdadero .also

&) Dearle afirma ()e la sol)cin al pro/lema de la li/ertad de la 4ol)ntad la p)ede proporcionar el determinismo psicolgicoDerdadero .also

0) Deg1n Dearle, la red de intencionalidad, c)ando es inconciente, es )na s)/clase de capacidades context)alesDerdadero .also

2) Deg1n Dearle, decir ()e )n agente tiene )n estado intencional inconciente 5 ()e 8ste inter4iene en s) comportamiento, significa ()e a. el individuo no es libre en sus acciones por!ue las mismas est n inconcientemente determinadas b. el individuo tiene un estado cerebral capaz de causar ese estado en forma conciente >) Dearle afirma ()e c)ando anali"amos Rla li/ertad de la 4ol)ntadR tenemos )n pro/lema- El mismo se de/e a la imposi/ilidad de conciliar la con4iccin 7)mana de la li/ertad como experiencia 5 la con4iccin cientfica de ()e todo 7ec7o ()e oc)rre de/e explicarse por s)s condiciones ca)salmente s)ficientesDerdadero .also

?) !a tesis del /acQgro)nd, prop)esta por Dearle, seala ()e Rc)al()ier estado intencional f)nciona como parte de )na malla de otros estados intencionalesRDerdadero .also

@) %)ede definirse Rmalla de la IntencionalidadR como: a. el significado !ue tienen nuestras acciones dentro de un contexto m s amplio b. el conjunto formado por los estados 4mentales2 intencionales c. el conjunto de destrezas" habilidades y h bitos !ue incluyen las acciones 1A) Dearle afirma Rn)estra concepcin de la realidad fsica no de6a l)gar simplemente para la li/ertad radicalR- Esto se de/e a ()e acepta ()e los

7ec7os9fenmenos psicolgicos son explica/les ca)salmente $5 de manera completa) mediante fenmenos fsicos9ne)ronalesDerdadero .also

%R'U!EFA E%IDPEF'!VGI<' $<BER<B!AND \ DEAR!E)


1) <7)rc7land considera ()e el pro/lema de inferir la existencia de estados mentales en otras personas, a partir de las cond)ctas o/ser4adas, es ()e se efect1an generali"aciones psicocond)ct)ales- El pro/lema se prod)ce por()e solo se o/ser4an las cond)ctas, pero n)nca se m)estra el otro aspecto de la conexin: los estados mentalesDerdadero .also

) Deg1n <7)rc7land, la sol)cin al pro/lema de la a)toconciencia la pro4ee: a. la teora de los datos sensoriales b. el dualismo de propiedades c. el conductismo filosfico d. la teora reticular #) Dearle considera ()e es posi/le resol4er el pro/lema de la identidad personal 5 para ello propone el principio de la conexinDerdadero .also &) Deg1n <7)rc7land, el principal pro/lema del arg)mento de la analoga es ()e lle4a a reali"ar generali"aciones psicocond)ct)ales mediante ind)cciones /asadas en )n 1nico casoDerdadero .also

0) Deg1n <7)rc7land, la a)toconciencia es: a. la captacin o percepcin permanente de la realidad interna b. el conocimiento permanentemente actualizado de la realidad externa c. la conciencia de poseer un cuerpo 2) El enfo()e tradicional de la a)toconciencia afirma ()e la mente se conoce a s misma en primer l)gar, de modo 1nico, 5 m)c7o me6or ()e lo ()e p)ede llegar a conocer el m)ndo externo- Deg1n <7)rc7land, esto sera errneo por()e la mente aprende so/re s misma a tra48s de )n proceso de desarrollo concept)al paralelo al proceso de comprensin del m)ndoDerdadero .also

>) Dearle afirma ()e la sensacin de ()e so5 exactamente el mismo indi4id)o a lo largo del tiempo, desde el p)nto de 4ista de la primera persona, se de/e a mi aptit)d de tener9prod)cir rec)erdos concientes so/re s)cesos anteriores de m mismoDerdadero .also

?) <)ando %7ineas Gage s)fri )n dao cere/ral prod)cido por )na /arra met.lica ()e le atra4es el cr.neo 5 perfor s) estr)ct)ra cere/ral, pas de ser )na persona ent)siasta, tra/a6adora 5 agrada/le, a ser 4icioso, desagrada/le 5 receloso- Dearle toma el caso para anali"ar )n criterio de contin)idad de n)estra identidad: a. la continuidad de la personalidad b. la continuidad espacio3temporal del cuerpo

c. la continuidad temporal de la estructura fsica @) Dearle anali"a la contin)idad espacio9temporal del c)erpo como )n criterio de identidad a. desde el punto de vista de la tercera persona b. desde una perspectiva mixta: primera y tercera persona a la vez c. desde el punto de vista de la primera persona 1A) !as dos dific)ltades ()e incl)5e el aspecto epistemolgico del %ro/lema mente9c)erpo son: a. el problema del libre albedrio b. el problema de las otras mentes c. el problema de la autoconciencia d. el problema de la especificacin de contenido

Potrebbero piacerti anche